Conwayy

You might also like

Download as pdf or txt
Download as pdf or txt
You are on page 1of 81

ε

ε ∈ (0, 1) such that Bd (x, ε) ⊂ O. Choose δ positive such that δ < 1+ε , then Bµ (x, δ) ⊂ Bd (x, ε). Similarly, if
δ
O is an open set in (S , µ) and δ is such that x ∈ Bµ (x, δ) ⊂ O then choose ε > 0 with ε ≤ 1−δ which implies
Bd (x, ε) ⊂ Bµ (x, δ). Since this can be done for any element x ∈ O, open sets in (S , d) are open in (S , µ) and
vice-versa.
The fact that exactly the open sets in (S , d) are open in (S , µ) leads to the statement about Cauchy sequences.
Assume that {xn }n is a Cauchy sequence in (S , d). To see that it is Cauchy in (S , µ), let ε > 0 be arbitrary
Chapter 7 but fixed. By the above there is a δ small enough that Bd (x, δ) ⊂ Bµ (x, ε) and δ depends only on ε, but not
on x ∈ S . Since {xn }n is Cauchy in (S , d) there is N ∈ N such that d(xm , xn ) < δ whenever m, n ≥ N and
therefore also µ(xm , xn ) < ε for m, n ≥ N. The opposite statement holds with a similar argument.

Compactness and Convergence in the Exercise 2. Find the sets Kn obtained in Proposition 1.2 for each of the following choices of G: (a) G is an
open disk; (b) G is an open annulus; (c) G is the plane with n pairwise disjoint closed disks removed; (d) G
is an infinite strip; (e) G = C − Z.
Space of Analytic Functions Solution. a) G is an open disk,
Let a ∈ C and r > 0 such that G = B(a, r). Set
!
1
Kn = B a, r −
7.1 The space of continuous functions C(G, Ω) n
t where a ball of negative radius is to be understood as the empty set. Depending on the size of r, a
Exercise 1. Prove Lemma 1.5 (Hint: Study the function f (t) = 1+t for t > −1.) S
finite number of Kn may be empty, not violating the condition Kn ⊂ int Kn+1 , and obviously G = ∞
n=1 .
d(s,t)
Solution. To show that µ : S × S → R, µ(s, t) = 1+d(s,t) is a metric we proof that µ satisfies the conditions
of a metric function. b) G is an open annulus,
d(s,t)
For s, t ∈ S the value µ(s, t) = 1+d(s,t) is well defined in the nonnegative real numbers because d is a metric. Let a ∈ C be the center of the annulus with radii r, R, 0 ≤ r < R < ∞ so that G = ann(a, r, R). Define
Also µ(s, t) = 0 if and only if d(s, t) = 0 and since d is a metric it follows that s = t. The metric d is
!
symmetric and therefore µ is also. It remains to show the triangle inequality. Let therefore s, t, u ∈ S . The 1 1
metric d satisfies the triangle inequality Kn = ann a, r + , R − ,
n n
d(s, u) ≤ d(s, t) + d(t, u). (7.1) again with the interpretation that an annulus with inner radius larger than the outer radius is the
t empty set.
The function f (t) = 1+t is a strictly increasing, concave function on the interval [0, ∞). Thus if d(s, u) ≤
max{d(s, t), d(t, u)}, then also µ(s, u) ≤ max{µ(s, t), µ(t, u)} and by nonnegativity of µ also d) G = {z ∈ C : |Imz| < 1},
µ(s, u) ≤ µ(s, t) + µ(t, u). For this open set G define
( )
If instead max{d(s, t), d(t, u)} < d(s, u) then 1
Kn = {z = x + iy ∈ C : x, y ∈ R, |x| ≤ n} ∩ z = x + iy ∈ C : x, y ∈ R|y| ≤ 1 − .
( ) n
1 1 1
< min , . S
1 + d(s, u) 1 + d(s, t) 1 + d(t, u) Although G is unbounded each Kn is bounded and closed and hence compact and G = ∞ n=1 Kn .

Together with equation (7.1) we have e) G = C − Z.


d(s, u) d(s, t) + d(t, u) Here we can define
µ(s, u) = ≤  n !C
1 + d(s, u) 1 + d(s, u)  [ 1 
Kn = {z ∈ C : |z| ≤ n} ∩  
 B ( j,  ,
d(s, t) d(t, u) d(s, t) d(t, u) n
= + ≤ + j=−n
1 + d(s, u) 1 + d(s, u) 1 + d(s, t) 1 + d(t, u)
an intersection of closed set, one of which is bounded, hence Kn is compact. Also Kn ⊂ int Kn+1 by
= µ(s, t) + µ(t, u), S
definition and G = ∞ n=1 .
which gives the triangle inequality.
Exercise 3. Supply the omitted details in the proof of Proposition 1.18.
To show that the metrics d and µ are equivalent on S , let O be on open set in (S , d) and let x ∈ O. There is

93 94
Solution. Not available. compact. To be more precise: For every point x in G, we can find an open neighborhood B(yi ; δ) for some

i = 1, . . . , k. Since fn → f pointwise, we have
Exercise 4. Let F be a subset of a metric space (X, d) such that F is compact. Show that F is totally
bounded. 
d( fn (yi ), f (yi )) < ∀n ≥ Ni .
3
Solution. Not available.
Set N = maxi Ni , then we have
Exercise 5. Suppose { fn } is a sequence in C(G, Ω) which converges to f and {zn } is a sequence in G which 
converges to a point z in G. Show lim fn (zn ) = f (z). d( fn (yi ), f (yi )) < ∀n ≥ N ∀i = 1, . . . , k. (7.4)
3
Solution. Not available. Finally, for an arbitrary x ∈ G and n ≥ N we have
Exercise 6. (Dini’s Theorem) Consider C(G, R) and suppose that { fn } is a sequence in C(G, R) which is   
d( fn (x), f (x)) ≤ d( fn (x), fn (yi )) + d( fn (yi ), f (yi )) + d( f (yi ), f (x)) < + + =
monotonically increasing (i.e., fn (z) ≤ fn+1 (z) for all z in G) and lim fn (z) = f (z) for all z in G where (7.2),(7.3),(7.4) 3 3 3
f ∈ C(G, R). Show that fn → f .
S where we choose yi such that |x − yi | < δ. This shows fn → f .
Solution. By Proposition 1.2 we can find a sequence {Kn } of compact subsets of G such that ∞ n=1 Kn . Thus, P Pn
WLOG we can assume that G is compact. Define gn = f − fn ∀n. Clearly gn is continuous ∀n, since f Exercise 8. (a) Let f be analytic on B(0; R) and let f (z) = ∞ n
n=0 an z for |z| < R. If fn (z) =
k
k=0 ak z , show

and fn are continuous by assumption. Since fn is monotonically increasing, gn is monotonically decreasing. that fn → f in C(G; C).
P
Since fn converges to f , gn converges pointwise to zero, that is (b) Let G = ann(0; 0, R) and let f be analytic on G with Laurent series development f (z) = ∞ n
n=−∞ an z . Put
Pn k
fn (z) = k=−∞ ak z and show that fn → f in C(G; C).

∀ > 0 ∃N ∈ N ∀x ∈ G : 0 ≤ gN (x) < . Solution. a) Let G = B(0; R). This result is proved with the Arzela-Ascoli Theorem. With the given function
2
f define an analytic function
Since gN is continuous, there is an open neighborhood B(x; r) around x (with r < ∞). For any y ∈ B(x; r) X∞

we have gN (y) < , that is g : G → C, g(z) := |ak |zk . (7.5)


k=0
∀ > 0 ∀x ∈ G ∃N ∈ N ∃B(x; r) : ∀y ∈ B(x; r) : gN (y) < . Let z ∈ G and define r := |z|+R
R) then z ∈ B(0; r). Let F = { fn }n . Then for this fixed z and for any n ∈ N
2 (<
we have
Since gN is monotonically decreasing, we surely get gn (y) <  for every n ≥ N and for every y ∈ B(x; r). n
X n
X
These open neighborhoods will cover G. Since G is compact, we can find finitely many x1 , . . . , xk such that | fn (z)| = | ak zk | ≤ |ak ||z|k = g(|z|) < ∞.
B(xi ; r) cover G for every value of . Note that the values of N can change the chosen center x of each k=0 k=0
those neighborhoods, say Ni . Therefore, let M = maxi Ni . Because of the monotonicity of gn , we have that Thus for a given z ∈ G, { fn (z) : fn ∈ F } is bounded and therefore has compact closure.
gn (y) <  ∀ n ≥ M in every open neighborhood B(xi ; r), that is, for every value in G. Hence, Next, we want to show equicontinuity at each point of G. Fix z0 ∈ G and let ε be positive. Let r > 0 be such
that B(z0 ; r) ⊂ G. With the function g(z) defined in equation (7.5) and the computations from above we have
∀ > 0 ∃M ∈ N, ∀n ≥ M ∀x ∈ G : | fn (x) − f (x)| < , for all w ∈ ∂B(z0 ; r) and for all n ∈ N
| fn (w)| ≤ g(|w|).
which shows fn → f .
The boundary of the disk with radius r is compact and so is g(∂B(z0 ; r)). Thus there is a positive number M
Exercise 7. Let { fn } ⊂ C(G, Ω) and suppose that { fn } is equicontinuous at each point of G. If f ∈ C(G, Ω) such that |g(∂B(z
 0 , r))| ≤ M.
and f (z) = lim fn (z) for each z then show that fn → f . Choose δ ∈ 0, min{ 2r , 2Mεr
} and let |z − z0 | < δ then as in the proof of Montel’s theorem the following
Solution. By Proposition 1.2 we can find a sequence {Kn } of compact subsets of G such that ∞
S estimate holds
n=1 Kn . Thus, I
WLOG we can assume that G is compact. Since fn is equicontiuous at each point of G, we have that 1 fn (w)(z − z0 )
| fn (z) − fn (z0 )| = | dw|
 2πi |w−z0 |=r (w − z)(w − z0 )
∀ > 0 ∃δ > 0 : d( fn (x), fn (y)) < whenever |x − y| < δ. (7.2) 2M
3 ≤ δ ≤ ε.
r
Taking the limit as n → ∞, we also get
By Arzela-Ascoli the sequence { fn } is normal in C(G) so there is a subsequence { fnk } that converges in C(G).

d( f (x), f (y)) < whenever |x − y| < δ. (7.3) Since f is analytic and { fn } is the sequence of partial sums of f , { fn } itself is an analytic Cauchy sequence
3 in C(G). Hence every subsequence converges to the same limit, which shows that fn → f in C(G). As a last
by the pointwise convergence ( f (z) = lim fn (z) ∀z ∈ G by assumption). As in the previous exercise, we step recall that H(G) is complete as a subspace of C(G) (Corollary 2.3) and { fn } ⊂ H(G) thus also the limit
can find finitely many points y1 , . . . , yk ∈ G such that the open neighborhoods B(yi ; δ) cover G, since G is function f ∈ H(G).

95 96
7.2 Spaces of analytic functions Solution. Not available.

Exercise 1. Let f, f1 , f2 , . . . be elements of H(G) and show that fn → f iff for each closed rectifiable curve Exercise 7. Suppose F is normal in H(G) and Ω is open in C such that f (G) ⊂ Ω for every f in F . Show
γ in G, fn (z) → f (z) uniformly for z in {γ}. that if g is analytic on Ω and is bounded on bounded sets then {g ◦ f : f ∈ F } is normal.

Solution. Not available. Solution. Not available.

Exercise 2. Let G be a region, letRa ∈ R, and suppose that f : [a, ∞] × G → C is a continuous func- Exercise 8. Let D = {z : |z| √
< 1} and show that F ⊂ H(D) is normal iff there is a sequence {Mn } of positive
∞ P
tion. Define the integral F(z) = a f (t, z) dt to be uniformly convergent on compact subsets of G if constants such that lim sup n Mn ≤ 1 and if f (z) = ∞ n
0 an z is in F then |an | ≤ Mn for all n.
Rb
limb→∞ a f (t, z) dt exists uniformly for z in any compact subset of G. Suppose that this integral does con- Solution. Not available.
verge uniformly on compact subsets of G and that for each t in (a, ∞), f (t, · ) is analytic on G. Prove that F
2πit
is analytic and Z ∞ k R 1) and for 0 < r < 1 let γr (t) = re , 0 ≤ t ≤ 1. Show that a sequence { fn } in
Exercise 9. Let D = B(0;
∂ f (t, z) H(D) converges to f iff γ | f (r) − fn (z)| |dz| → 0 as n → ∞ for each r, 0 < r < 1.
F (k) (z) = dt r

a ∂zk
Solution. ⇒: Assume limn→∞ fn (z) = f (z) ∀z ∈ D. Then
Solution. Not available. Z Z
Exercise 3. The proof of Montel’s Theorem can be broken up into the following sequence of definitions and | f (z) − fn (z)| |dz| ≤ sup | f (z) − fn (z)| |dz|
γr z∈γr γr
propositions: (a) Definition. A set F ⊂ C(G, C) is locally Lipschitz if for each a in G there are constants
= sup | f (z) − fn (z)|· 2πr, ∀0 < r < 1.
M and r > 0 such that | f (z) − f (a)| ≤ M|z − a| for all f in F and |z − a| < r. (b) If F ∈ C(G, C) is locally γr =re2πit ,0≤t≤1 z∈γr
Lipschitz then F is equicontinuous at each point of G. (c) If F ⊂ H(G) is locally bounded then F is locally
Lipschitz. But supz∈γr | f (z) − fn (z)| → as n → ∞ since limn→∞ fn (z) = f (z) ∀ z ∈ D. Therefore,
Z
Solution. Not available.
| f (z) − fn (z)| |dz| → 0
Exercise 4. Prove Vitali’s Theorem: If G is a region and { fn } ⊂ H(G) is locally bounded and f ∈ H(G) that γr

has the property that A = {z ∈ G : lim fn (z) = f (z)} has a limit point in G then fn → f . as n → ∞ forR each r, 0 < r < 1.
Solution. The sequence { fn } is a locally bounded sequence in H(G) and by Montel’s Theorem then { fn } is ⇐: Assume γ | f (z) − fn (z)| |dz| → 0 as n → ∞ for each r, 0 < r < 1. Let K be an arbitrary compact subset
r

normal, so there is a subsequence { fnk } ⊂ { fn } that converges to f in H(G). of D = B(0; 1) and choose 0 < r < 1 such that K ⊂ B(0; r) and denote the shortest distance between an
Suppose that fn 9 f in H(G) then there must be a compact set K ⊂ G the convergence is not uniform on arbitrary point in K and an arbitrary point on ∂B by δ > 0. By the assumption, we have
K. In other words there must be an ε > 0 so that for all n ∈ N there is zn ∈ K with | fn (zn ) − f (zn )| ≥ ε. By Z
compactness of K extract a subsequence of {zn }, say {znm } that converges to a point z0 ∈ K. | f (z) − fn (z)| |dz| < 2πδ. (7.6)
The sequence { fn } is locally bounded, so in particular the subsequence { fnm } with indices corresponding ∂B

to {znm } is, and again by Montel’s Theorem now applied to the subsequence and again the completeness Let a ∈ K, then by Cauchy’s Integral formula, we have
of H(G) there is an analytic function g such that fnm → g in H(G). On the set A of points of pointwise Z
convergence fn (z) → f and fn (z) → g. 1 f (z) − fn (z)
f (a) − fn (a) = dz.
Now G is a region and A has a limit point in G so by the Identity Theorem (Chapter 4, Corollary 3.8) 2πi ∂B z−a
already f = g on G which gives a contradiction on the set K and the point z0 because
Thus, Z Z
1 | f (z) − fn (z)| 1 2πδ
| fnm (znm ) − f (znm )| → |g(z0 ) − f (z0 )| ≥ ε. | f (a) − fn (a)| ≤ |dz| ≤ | f (z) − fn (z)| |dz| < = .
2π ∂B |z − a| 2πδ ∂B (7.6) 2πδ
Hence we can conclude that fn → f in H(G).
Hence, fn (a) → f (z) uniformly ∀a ∈ K. Since it converges uniformly on all compact subsets of D, Proposi-
Exercise 5. Show that for a set F ⊂ H(G) the following are equivalent conditions: tion 1.10 b) yields that { fn } converges to f ∀ z ∈ D.
(a) F is normal;
Exercise 10. Let { fn } ⊂ H(G) be a sequence of one-one functions which converge to f . If G is a region,
(b) For every  > 0 there is a number c > 0 such that {c f : f ∈ F } ⊂ B(0; ) (here B(0; ) is the ball in
show that either f is one-one or f is a constant function.
H(G) with center at 0 and radius ).
Solution. Assume { fn } ⊂ H(G) is a sequence of one-one functions which converge to f where G is a region
Solution. Not available.
(open and connected). Suppose f is not the constant function. Then, we have to show that f is one-one.
Exercise 6. Show that if F ⊂ H(G) is normal then F 0 = { f 0 : f ∈ F } is also normal. Is the converse true? Choose an arbitrary point z0 ∈ G and define the sequence gn (z) = fn (z) − fn (z0 ). Clearly, the sequence {gn }
Can you add something to the hypothesis that F 0 is normal to insure that F is normal? converges to g(z) = f (z) − f (z0 ) on the open connected set G\{z0 }, which is again a region. In addition, we

97 98
have that gn never vanishes on G\{z0 }, since fn are assumed to be one-one functions. b) Let G be a region and M be a fixed constant. Let F be the family of all functions f in H(G) such that
Therefore, the sequence {gn } satisfies the conditions of the Corollary 2.6 resulting from Hurwitz’s Theorem, "
where the region is G\{z0 }. Thus, either g ≡ 0 or g never vanishes (g has no zeros). Since g(z) = f (z) − f (z0 ) | f (z)|2 dx dy ≤ M. (7.8)
is not identically zero on G\{z0 }, we must have g(z) = f (z) − f (z0 ) has no zeros in G\{z0 }. This implies G

To show that F is normal.


f (z) , f (z0 ) ∀z ∈ G\{z0 }. According to Montel’s Theorem, it suffices to show that F is locally bounded, that is ∃ r > 0 such that
Since z0 was an arbitrary point in G, we have shown that f is one-one on G. sup{| f (z)| : |z − a| < r, f ∈ F } < ∞ (p. 153). We will prove this fact by contradiction:
Assume F is not locally bounded. Then there is a compact set K ⊂ G such that sup{| f (z)| : z ∈ K, f ∈ F } =
Exercise 11. Suppose that { fn } is a sequence in H(G), f is a non-constant function, and fn → f in H(G). ∞. That is, there is a sequence { fn } in F such that sup{| fn (z)| : z ∈ K} ≥ n. Therefore, ∃ zn ∈ K such that
Let a ∈ G and α = f (a); show that there is a sequence {an } in G such that: (i) a = lim an ; (ii) fn (an ) = α n
for sufficiently large n. | fn (zn )| ≥ . (7.9)
2
Solution. Not available. Every sequence has a convergence subsequence, say {znk }, with znk → z0 ∈ K since K is compact. WLOG,
we write {znk } = {zn }. Clearly z0 ∈ G since z0 ∈ K (K ⊂ G), hence ∃ R > 0 such that B(z0 ; R) ⊂ G (since
Exercise 12. Show that lim tan nz = −i uniformly for z in any compact subset of G = {z : Im z > 0}.
G is open). If we pick n large enough, we can get zn ∈ B(z0 ; R2 ) (a picture might help). In addition, we can
Solution. Not available. find an R̂ > 0 such that B(zn ; R̂) ⊂ B(z0 ; R2 ) because B(z0 ; R2 ) is open. Apply part a) to the ball B(zn ; R̂) to
Exercise 13. (a) Show that if f is analytic on an open set containing the disk B̄(a; R) then obtain:
Z 2π Z R̂ "
1
Z 2π Z R 1 2 1 1
| f (a)|2 ≤ | f (a + reiθ )|2 r dr dθ. | fn (zn )|2 ≤ fn (zn + reiθ ) r dr dθ ≤ | fn (z)|2 dx dy ≤ M < ∞.
2 2
πR2 0 0
πR̂ 0 0 B(zn ;R̂)⊂G πR̂ G (7.8) πR̂3

be a region and let M be a fixed positive constant. Let F be the family of all functions f in H(G)
(b) Let G ! So r
such that G | f (z)|2 dx dy ≤ M. Show that F is normal. M1
| fn (zn )| ≤ < ∞.
π R̂
it
Solution. a) Let B̄(a; R) ⊂ G and f : G → C be analytic. Let γ(t) = a + re for 0 ≤ t ≤ 2π, then by But by (7.9) we have
Cauchy’s Integral Formula we have n
| fn (zn )| ≥
.
Z Z 2π 2
1 g(w) 1
g(a) = dw = f (a + reit ) dt. This gives a contradiction if we let n → ∞. Thus, F is locally bounded and therefore normal.
2πi γ w − a 2π 0

Thus, letting g(z) = f (z)2 , we obtain 7.3 Spaces of meromorphic functions


Z 2π
2 1 h
it
i2
f (a) = f (a + re ) dt. Exercise 1. Prove Proposition 3.3.
2π 0
Solution. a) Let a ∈ C and r > 0. To show: There is a number δ > 0 such that B∞ (a; δ) ⊂ B(a; r). Let
Multiply by r, we get R > 0 such that B(a; r) ⊂ B(0; R) and B∞ (a; δ) ⊂ B(0; R).
Z 2π Claim: Pick
1 h i2
2r
f 2 (a)r = f (a + reiθ ) r dθ (7.7) δ= √
2π 0
p
1 + R2 1 + |a|2
and then integrate with respect to r yields to obtain
Z R Z R Z 2π h Z 2π Z R h B∞ (a; δ) ⊂ B(a; R).
R2 1 i2 1 i2
f 2 (a)r dr = f 2 (a) = f (a + reiθ ) r dθ dr = f (a + reiθ ) r dr dθ Proof of the claim: Let z ∈ B∞ (a; δ). This is equivalent to
0 2 (7.7) 0 2π 0 2π 0 0
2|z − a|
which implies p p <δ
1
Z 2π Z R h i2 1 + |z|2 1 + |a|2
f 2 (a) = f (a + reiθ ) r dr dθ. 2|z − a| 2r
πR2 0 0 ⇐⇒ p p < √ p
So Z Z 1 + |z|2 1 + |a|2 1 + R2 1 + |a|2
2π R r
1 2
p
| f (a)|2 ≤ f (a + reiθ ) r dr dθ. ⇐⇒ |z − a| < √ 1 + |z|2
πR2 0 0 1 + R2
99 100
which implies (z ∈ B∞ (a; δ) ⇒ z ∈ B(0; R), that is |z| < R) Proof of the claim:
r √ C∞ − K = C∞ − B(0; r)
|z − a| < √ 1 + R2
1 + R2 = {C − B(0; r)} ∪ {∞}
⇐⇒ |z − a| < r. = {C − {z ∈ C : |z| ≤ r}} ∪ {∞}
  r 
So z ∈ B(a; r). Therefore


 

 4 



q⊂  C− z ∈ C : |z| ≤ − 1

∪ {∞}
B∞ (a; δ) ⊂ B(a; r). r> 4
−1

 
 δ2 

δ2

b) Let δ > 0 and a ∈ C. To show: There is a number r > 0 such that B(a; r) ⊂ B∞ (a; δ).  r 

 4 

Claim: Pick r < 2δ to obtain
 
= z ∈ C : |z| > 2
− 1 ∪ {∞}
B(a; r) ⊂ B∞ (a; δ).

 δ 

 r 
Proof of the claim: Let z ∈ B(a; r). This is equivalent to


 4 


=  z∈C: 2
− 1 < |z| ∪ {∞}

 δ 

|z − a| < r  
δ


 2 


⇒δ |z − a| < = 
 z∈C: p < δ
 ∪ {∞}
r< 2 2
 1 + |z|2 

δ = {z ∈ C : d(∞, z) < δ} ∪ {∞}


⇐⇒ |z − a| < · 1 = B∞ (∞; δ).
2
δp p
√ ⇒√ |z − a| < 1 + |z|2 1 + |a|2 Therefore,
1+|z|2 1+|a|2 2
1≤ C∞ − K ⊂ B∞ (∞; δ).
2|z − a| d) Let K ⊂ C where K is compact. To show: There is a number δ > 0 such that B∞ (∞; δ) ⊂ C∞ − K. Let
⇐⇒ p p <δ
1 + |z|2 1 + |a|2 B(0; r) such that B(0; r) ⊃ K where r > 0. Clearly
⇐⇒ d(z, a) < δ.
C∞ − B(0; r) ⊂ C∞ − K.
So z ∈ B∞ (a; δ). Therefore, So it suffices to show: There is a number δ > 0 such that
B(a; r) ⊂ B∞ (a; δ).
B∞ (∞; δ) ⊂ C∞ − B(0; r)
q set K ⊂ C such that C∞ − K ⊂ B∞ (∞; δ).
c) Let δ > 0. To show: There is a compact
Claim: Choose K = B(0; r) where r > 4
− 1 to obtain for a given r > 0.
δ2
Claim: Choose δ ≤ √2 to obtain
r2 +1
C∞ − K ⊂ B∞ (∞; δ). B∞ (∞; δ) ⊂ C∞ − B(0; r)
and hence
(Clearly K is compact and r > 0 since δ ≤ 2.)
B∞ (∞; δ) ⊂ C∞ − K.
Proof of the claim:
B∞ (∞; δ) = {z ∈ C : d(∞, z) < δ} ∪ {∞}
 


 2 


= 
 z∈C: p < δ
 ∪ {∞}
 1 + |z|2 
 


 2 2 


⊂ 
 z∈C: p < √  ∪ {∞}

δ≤ √ 2  1 + |z|2 r2 + 1 
r2 +1

= {z ∈ C : r ≤ |z|}} ∪ {∞}
= {C ∪ {z ∈ C : |z| < r} ∪ {∞}
= C∞ − {z ∈ C : |z| < r}
= C∞ − B(0; r).

101 102
Therefore, We also have
B∞ (∞; δ) ⊂ C∞ − B(0; r). |h(z)| ≤ 1 ∀z ∈ D
Exercise 2. Show that if F ⊂ M(G) is a normal family in C(G, C∞ ) then µ(F ) is locally bounded. since h : D → D. Thus, the hypothesis of Schwarz’s Lemma are satisfied, and hence, we get
Solution. Not available. |h0 (0)| ≤ 1.

We have
7.4 The Riemann Mapping Theorem h i0  0
h0 (z) = g( f (g−1 (z))) = g0 ( f (g−1 (z))) f 0 (g−1 (z)) g−1 (z)
Exercise 1. Let G and Ω be open sets in the plane and let f : G → Ω be a continuous function which 1
is one-one, onto, and such that f −1 : Ω → G is also continuous (a homeomorphism). Suppose {zn } is = g0 ( f (g−1 (z))) f 0 (g−1 (z))
g0 (g−1 (z))
a sequence in G which converges to a point z in ∂G; also suppose that w = lim f (zn ) exists. Prove that
w ∈ ∂Ω. where the last step follows from Proposition 2.20 provided g0 (g−1 (z)) , 0. So
Solution. Not available. 1
h0 (0) = g0 ( f (g−1 (0))) f 0 (g−1 (0))
Exercise 2. (a) Let G be a region, let a ∈ G and suppose that f : (G − {a}) → C is an analytic function g0 (g−1 (0))
such that f (G − {a}) = Ω is bounded. Show that f has a removable singularity at z = a. If f is one-one, 1 1
= g0 ( f (a)) f 0 (a) = g0 (a) f 0 (a) 0 = f 0 (a)
show that f (a) ∈ ∂Ω. g0 (a) g (a)
(b) Show that there is no one-one analytic function which maps G = {z : 0 < |z| < 1} onto an annulus
Ω = {z : r < |z| < R} where r > 0. (g0 (a) > 0 by assumption). Therefore
|h0 (0)| = | f 0 (a)| ≤ 1.
Solution. Not available.
Thus, we have shown that
Exercise 3. Let G be a simply connected region which is not the whole plane and suppose that z̄ ∈ G | f 0 (a)| ≤ 1.
whenever z ∈ G. Let a ∈ G ∩ R and suppose that f : G → D = {z : |z| < 1} is a one-one analytic function
with f (a) = 0, f 0 (a) > 0 and f (G) = D. Let G+ = {z ∈ G : Im z > 0}. Show that f (G+ ) must lie entirely Exercise 6. Let G1 and G2 be simply connected regions neither of which is the whole plane. Let f be a
above or entirely below the real axis. one-one analytic mapping of G1 onto G2 . Let a ∈ G1 and put α = f (a). Prove that for any one-one analytic
map h of G1 into G2 with h(a) = α it follows that |h0 (a)| ≤ | f 0 (a)|. Suppose h is not assumed to be one-one;
Solution. Not available. what can be said?
Exercise 4. Find an analytic function f which maps {z : |z| < 1, Re z > 0} onto B(0; 1) in a one-one fashion. 1−1
Solution. Define the function F(z) = f −1 (h(z)). This is well-defined since f : G1 −−−−−−→ G2 and h :
onto
Solution. Not available. 1−1
G1 −−−−−−→ G2 . Clearly F(z) is analytic since f and h are analytic, F(z) is one-one and F : G1 → G1 by
Exercise 5. Let f be analytic on G = {z : Re z > 0}, one-one, with Re f (z) > 0 for all z in G, and f (a) = a construction. We have
for some real number a. Show that | f 0 (a)| ≤ 1. F(a) = f −1 (h(a)) = f −1 (α) = a.
Solution. Clearly G is a simply connected region (not the whole plane). Let a ∈ G. Then, there is a unique Thus by Exercise 5, we get
analytic function g : G → D having the properties |F 0 (a)| ≤ 1.
a) g(a) = 0 and g0 (a) > 0
We have
b) g is 1-1
c) g(G) = D  0 1
1−1 F 0 (z) = f −1 (h(z)) = f 0 (h(z))h0 (z) = h0 (z)
(by the Riemann Mapping Theorem). Since, we have f : G −−−−−−→ G (since Re f (z) > 0) and g : f 0 ( f −1 (h(z)))
1−1 1−1
−−−−−−→ D so g−1 : D −−−−−−→ G we can define h(z) = g( f (g−1 (z))). Clearly h(z) is analytic and one- where the last step follows by Proposition 2.20 provided f 0 ( f −1 (h(z))) , 0. So
onto onto
one, since f and g are analytic and one-one, and we have 1 h0 (a) h0 (a)
F 0 (a) = h0 (a) = =
h(D) = D f 0 ( f −1 (h(a))) f 0 ( f −1 (α)) f 0 (a)
which is well-defines since f 0 (a) , 0 by assumption (f is one-one). Since
by construction. We have
|h0 (a)|
h(0) = g( f (g−1 (0))) = g( f (a)) = g(a) = 0. |F 0 (a)| = ≤1
g−1 (0)=a f (a)=a | f 0 (a)|
103 104
gives since h satisfies the properties of the Riemann Mapping Theorem, too. Thus,
|h0 (a)| ≤ | f 0 (a)|.
g(z) − α
f (z) = h(z) = g3 (g2 (z)) = eiθ g2 (z) = eiθ g1 (g(z)) = eiθ .
If h is not one-one, then we get the same result, since I do not need the assumption that F(z) has to be 1 − ᾱg(z)
one-one for exercise 5.
So
Exercise 7. Let G be a simply connected region and suppose that G is not the whole plane. Let ∆ = {ξ :
1 − ᾱg(z) = eiθ g(z) − αeiθ
|ξ| < 1} and suppose that f is an analytic, one-one map of G onto ∆ with f (a) = 0 and f 0 (a) > 0 for some
point a in G. Let g be any other analytic, one-one map of G onto ∆ and express g in terms of f . ⇐⇒ f (z) − ᾱg(z) f (z) = eiθ g(z) − αeiθ
⇐⇒ (eiθ + ᾱ f (z))g(z) = f (z) + αeiθ
Solution. Let a ∈ G such that f (a) = 0 and f 0 (a) > 0. Since g is one-one mapping from G onto ∆, we
f (z) + αeiθ
clearly have g(a) = α, where α ∈ ∆ (α does not have to be zero). Next, define ⇐⇒ g(z) = iθ .
e + ᾱ f (z)
z−α
g1 (z) = . Exercise 8. Let r1 , r2 , R1 , R2 , be positive numbers such that R1 /r1 = R2 /r2 ; show that ann(0; r1 , R1 ) and
1 − ᾱz
ann(0; r2 , R2 ) are conformally equivalent.
Clearly, g1 is a Möbius transformation mapping ∆ to ∆ and in addition g1 is one-one and analytic. Now
Solution. The function f : C → C; f (z) = rr21 z is analytic and it maps the annulus ann(0; r1 , R1 ) bijectively
define
into ann(0; r2 , R2 ). The inner boundary {z : |z| = r1 } of the domain is mapped to the inner boundary of
g2 (z) = g1 (g(z)).
ann(0; r2 , R2 ). For the outer boundary note that by assumption R1 = Rr22r1 , so if |z| = R1 then | f (z)| = rR2 r11 =
Then, g2 is one-one and analytic and maps G onto ∆. In addition, we have R2 . The inverse function is f −1 (z) = rr12 z, an analytic and non-constant function also. Therefore the result is
proved.
g2 (a) = g1 (g(a)) = g1 (α) = 0. (7.10)
Exercise 9. Show that there is an analytic function f defined on G =ann(0; 0, 1) such that f 0 never vanishes
But g02 (a)
need not necessarily satisfy g02 (a) > 0. However, g02 (a) , 0 ∀z ∈ G since g2 is one-one. Therefore, and f (G) = B(0; 1).
define the rotation
Solution. Not available.
g3 (z) = eiθ z
for θ ∈ [0, 2π). Clearly g3 maps ∆ onto ∆. g3 is one-one and analytic. Finally, let
7.5 The Weierstrass Factorization Theorem
h(z) = g3 (g2 (z)). Q Q
Exercise 1. Show that (1 + zn ) converges absolutely iff (1 + |zn |) converges.
Also h is one-one, analytic and maps G onto ∆. We have Q P
Solution. Assume Re(zn ) > −1. Then the product (1 + zn ) converges absolutely iff ∞ k=1 zn converges
P∞ P
h(a) = g3 (g2 (a)) = g3 (0) = 0. absolutely (by Corollary 5.6 p. 166 applied to wn = 1 + zn ). k=1 zn converges absolutely iff ∞ n=1 |zn |
(7.10) converges (by definition).
P∞ P∞
Claim: n=1 |zn | converges iff n=1 log(1 + |zn |) converges.
P Q
And, Then ∞ n=1 log(1+|zn |) converges iff (1+|zn |) converges (by Proposition 5.2 p.165 applied to wn = 1+|zn |).
Q Q
Thus, ones we have proved the claim, we obtain: (1 + zn ) converges absolutely iff (1 + |zn |) converges.
h0 (z) = (g3 (g2 (z)))0 = (g3 (g1 (g(z))))0 = g03 (g1 (g(z)))g01 (g(z))g0 (z) Proof of the claim: Let xn = |zn | for convenience. Clearly xn ≥ 0 ∀n ∈ N.
(1 − ᾱg(z))· 1 − (g(z) − α)· (−ᾱ) 0 P P
⇒: Assume |zn | converges, that is xn converges. Therefore, we have xn → 0. So given  > 0, we have
= eiθ g (z).
(1 − ᾱg(z))2
0 ≤ log(1 + |xn |) ≤ (1 + )xn
So,
(1 − ᾱα) − 0 0 1 − |α| 2
1 for sufficiently large n where the last inequality follows since limz→0 log(1+z)
z = 1 by Exercise 2. By the
h0 (a) = eiθ g (a) = eiθ g0 (a) = eiθ g0 (a). P P
comparison test, log(1 + xn ) converges, that is log(1 + |zn |) converges.
(1 − ᾱα)2 (1 − |α|2 )2 1 − |α|2 P P
⇐: Assume log(1 + |zn |) converges, that is log(1 + xn ) converges. Therefore, we have log(1 + xn ) → 0
Clearly, we can choose a θ ∈ [0, 2π) such that h0 (a) > 0 (depending on the sign of g0 (a). Recall g0 (a) , 0 and therefore xn → 0. So given  > 0, we have
1 0 iθ
since g is one-one and α ∈ ∆, so 1−|α| 2 > 0). For example: If g (a) < 0, then pick θ = π, so e = −1 or
θ = −Arg(g0 (a)) will work, too. (1 − )xn ≤ log(1 + xn )
By uniqueness of the map f ( f satisfies the properties of the Riemann Mapping Theorem), we have log(1+z)
for sufficiently large n where the last inequality follows since limz→0 z = 1 by Exercise 2. By the
P P
f (z) = h(z) comparison test, xn converges, that is |zn | converges.
P∞ P∞
Therefore, n=1 |zn | converges iff n=1 log(1 + |zn |) converges.

105 106
Exercise 2. Prove that limz→0 log(1+z)
z = 1. converges in H(B(0; 1)) and that |B(z)| ≤ 1. What are the zeros of B? (B(z) is called a Blaschke Product.)
P
(c) Find a sequence {an } in B(0; 1) such that (1 − |an |) < ∞ and every number eiθ is a limit point of {an }.
Solution. First note that,
log(1 + z) Solution. Not available.
f (z) =
z Q∞ 1
Exercise 5. Discuss the convergence of the infinite product n=1 n p for p > 0.
has a removable singularity at z = 0, since
Solution. Not available.
log(1 + z) Qh i
lim z f (z) = lim z = lim log(1 + z) = log(1) = 0 Exercise 6. Discuss the convergence of the infinite products 1 + ni and
Q
1 + ni .
z→0 z→0 z z→0
 
(by Theorem 1.2 Chapter 5 p. 103). We know Q∞ i
Solution. First consider n=1 1 + n . We claim that this product does not converge.

q  
zk
Q
Let zn := 1 + ni = rn eiθn with rn = 1 + n12 and θ = arctan 1n . If there is z = reiθ such that ∞
X
log(1 + z) = (−1)k+1 n=1 zn = z,
p.165 k Q∞ P∞
k=1 then n=1 rn = r and n=1 θn = θ.
Since arctan(x) ≥ 12 for all values x ∈ (0, 1] the estimate
if |z| < 1. Therefore, x
∞ ∞ ! ∞
P∞ k+1 zk ∞ ∞ ∞
X X 1 1X1
log(1 + z) k=1 (−1) k
X zk−1 X zk−1 X zk θn = arctan ≥
f (z) = = = (−1)k+1 =1+ (−1)k+2 =1+ (−1)k n 2 n n
z z k=1
k k=2
k k=1
k +1 n=1 n=1 1

is valid. The right hand side does not converge, therefore the angles do not converge and there is no z ∈ C
where the sum is an analytic function. Hence,
  with the desired properties (note that rn > 1 for all n, thus z , 0).
∞ ∞ ∞
 X zk  X (limz→0 z)k X
(−1)k (−1)k
q
lim f (z) = lim 1 +  = 1 + =1+ 0=1
z→0 z→0 k+1 k+1 The second infinite product is a product of real numbers and zn = |1 + ni | = 1 + n12 . The estimate
k=1 k=1 k=1
 2
1 1 1
and therefore 1 + n2 ≤ 1 + n2 implies that 1 ≤ zn ≤ 1 + n2 . Thus for any integer N ∈ N
log(1 + z)
limz→0 = 1. N
Y N
Y N
Y 1
!
z 1≤ zn ≤ 1+
Exercise 3. Let f and g be analytic functions on a region G and show that there are analytic functions n=1 n=1 n=1
n2
f1 , g1 , and h on G such that f (z) = h(z) f1 (z) and g(z) = h(z)g1 (z) for all z in G; and f1 and g1 have no where the last term is absolutely convergent by an example in class. Therefore the sequence of the partial
common zeros. QN
products n=1 zn is an increasing, bounded sequence that converges in the real numbers. With log(zn ) > 0
Solution. Let f and g be analytic functions on the region G. Let {a j } be the zeros of f with multiplicity n j . for all n, the comparison with the sum of logarithms (Proposition 5.4) gives absolute convergence.
Let {b j } be the zeros of g with multiplicity ñ j . Q  
Exercise 7. Show that ∞ 1 1
n=2 1 − n2 = 2 .
If f and g have no common zeros, let h = 1. Then f1 = f and g1 = g. Clearly f1 , g1 and h are analytic on
G such that f (z) = h(z) f1 (z) and g(z) = h(z)g1 (z) for all z in G and f1 and g1 have no common zeros. Solution. Consider
Otherwise let {z j } be the set of common zeros of f and g with multiplicity m j = min(n j , ñ j ). Then {z j } form n
Y 1
! Y n
1
!
1
! Y n !
k−1 k+1
!
a sequence of distinct points in G with no limit points on G (since the zeros are isolated). Then there is an Pn = 1− = 1 − 1 + =
k=2
k2 k=2
k k k=2
k k
analytic function h defined on G whose only zeros are at the points z j with multiplicity m j (by Theorem 5.15  n ! !  n " ! !#
p. 170). Let f1 = hf and g1 = gh , then clearly f1 and g1 are analytic functions on G such that f (z) = h(z) f1 (z) 

 Y k−1 k+1  
 

 X k−1 k+1  

= exp  log  = exp  log 
and g(z) = h(z)g1 (z) for all z in G. (since f1 and g1 have removable singularities at the z j ’s by construction; 
 k k   
 k k 

k=2 k=2
the multiplicity of z j ’s in f and g is greater or equal than the multiplicity of z j ’s in h). In addition, we have  n   n 

 X 
 
 X 

that f1 and g1 have no common zeros by construction. =
 
exp  log(k − 1) − log(k) 
  
+ exp  log(k + 1) − log(k) 


 
 
 

Exercise 4. (a) Let 0 < |a| < 1 and |z| ≤ r < 1; show that 
k=2
 
k=2

= exp log(1) − log(n) + exp − log(2) + log(n + 1)
a + |a|z 1+r ( !)
≤   n+1 n+1
(1 − āz)a 1−r = exp − log(2n) + log(n + 1) = exp log = .
P 2n 2n
(b) Let {an } be a sequence of complex numbers with 0 < |an | < 1 and (1 − |an |) < ∞. Show that the infinite
product So,
∞ !
Y∞
|an | an − z
! Y 1 n+1 1
1− = lim Pn = lim = .
B(z) = n2 n→∞ n→∞ 2n 2
n=1
an 1 − ān z n=2

107 108
Q Q
Exercise 8. For which values of z do the products (1 − zn ) and (1 + z2n ) converge? Is there an open iff A has a limit point in G.
set G such that the product converges uniformly on each compact subset of G? If so, give the largest such (d) Let a ∈ G and J = J({a}). Show that J is a maximal ideal.
open. (e) Show that every maximal ideal in H(G) is a prime ideal.
Q (f) Give an example of an ideal which is not a prime ideal.
Solution. Consider the infinite product ∞ n
n=1 (1 − z ). By definition and Corollary 5.6 this converges abso-
P
lutely if and only if ∞ n=1 −z n
converges absolutely. From Calculus it is known that the convergence is also Solution. Not available.
uniform for |z| ≤ r < 1 with r ∈ [0, 1).
If z is an nth root of unity for a positive integer n, then one of the factors in the infinite product becomes 0 Exercise 12. Find an entire function f such that f (n + in) = 0 for every integer n (positive, negative or
and the product converges to 0, but not absolutely according to Definition 5.5. If there is no n such that z zero). Give the most elementary example possible (i.e., choose the pn to be as small as possible).
is nth root of unity then no factor of the infinite product equals zero and there is a subsequence {(1 − znk }k Solution. Not available.
Q
such that |1 − znk | > 1.5. We conclude that ∞ n
n=1 (1 + z ) does not converge.
S th
With the denseness of n∈N {z ∈ C : z is n root of unity} in the unit sphere it follows that the maximal set G Exercise 13. Find an entire function f such that f (m + in) = 0 for all possible integers m, n. Find the most
such that the convergence is uniform on compact subsets is the unit disk. elementary solution possible.
Q
The behavior of ∞ 2n 2
n=1 (1 + z ) is almost the same as the previous one. Substitute w := z , then the conver-
gence is uniform for |w| ≤ r < 1, so |z| ≤ sqrtr < 1. Also if |z| -and hence |w|- is greater than 1, then the Solution. Not available.
infinite product diverges. Similar to the previous case one factor equals 0 if z is a 2nth root of −1.
The maximal region G such that the convergence is uniform on compact sets is again the open unit disk.
7.6 Factorization of the sine function
Exercise 9. Use Theorem 5.15 to show there is an analytic function f on D = {z : |z| < 1} which is not Q∞ h i
4z2
analytic on any open set G which properly contains D. Exercise 1. Show that cos πz = n=1 1 − (2n−1)2
.

Solution. Not available. Solution. We know by the double-angle identity of sine sin(2z) = 2 sin(z) cos(z) (this is proved
 easily by
Q z2
Q using the definition) or sin(2πz) = 2 sin(πz) cos(πz). Since, we know sin(πz) = πz ∞
n=1 1 − n2 , we obtain
Exercise 10. Suppose G is an open set and { fn } is a sequence in H(G) such that f (z) = fn (z) converges
in H(G). (a) Show that   sin(2πz) = 2 sin(πz) cos(πz)
X∞  ∞ ! ∞ !
 0 Y  Y 4z2 Y z2
 fk (z) fn (z) ⇐⇒ 2πz 1 − 2 = 2πz 1 − 2 cos(πz)
k=1 n,k n=1
n n=1
n
converges in H(G) and equals f 0 (z). (b) Assume that f is not the identically zero function and let K be a Y∞
4z 2
!Y∞
4z2
! Y∞
z2
!
compact subset of G such that f (z) , 0 for all z in K. Show that ⇐⇒ 2πz 1− 1 − = 2πz 1 − cos(πz)
m=1
(2m)2 m=1 (2m − 1)2 n=1
n2

f 0 (z) X fn0 (z)
= where the last statement follows by splitting the product into a product of the even and odd terms (rear-
f (z) n=1
fn (z) rangement of the terms is allowed). Hence
and the convergence is uniform over K. ∞
Y 4z2 Y
! ∞
4z2
! Y ∞
z2
!
2πz 1− 2
1− 2
= 2πz 1 − 2 cos(πz)
Solution. Not available. n=1
(2n) n=1
(2n − 1) n=1
n
∞ ! ∞ ! ∞ !
Exercise 11. A subset J of H(G), G a region, is an ideal iff: (i) f and g in J implies a f + bg is in J for Y z2 Y 4z2 Y z2
⇐⇒ 2πz 1− 2 1− = 2πz 1 − 2 cos(πz).
all complex numbers a and b; (ii) f in J and g any function in H(G) implies f g is in J. J is called a n=1
n n=1
(2n − 1) 2
n=1
n
proper ideal if J , (0) and J , H(G); J is a maximal ideal if J is a proper ideal and whenever L is an
ideal with J ⊂ L then either L = J or L = H(G); J is a prime ideal if whenever f and g ∈ H(G) and Thus,
∞ !
f g ∈ J then either f ∈ J or g ∈ J. If f ∈ H(G) let Z( f ) be the set of zeros of f counted according to Y 4z2
cos(πz) = 1− .
their multiplicity. So Z((z − a)3 ) = {a, a, a}. If S ⊂ H(G) then Z(S) = ∩{Z( f ) : f ∈ S}, where the zeros (2n − 1)2
n=1
are again counted according to their multiplicity. So if S = {(z − a)3 (z − b), (z − a)2 } then Z(S) = {a, a}.
(a) If f and g ∈ H(G) then f divides g (in symbols, f |g if there is an h in H(G) such that g = f h. Show that Exercise 2. Find a factorization for sinh z and cosh z.
f |g iff Z ⊂ Z(g).
(b) If S ⊂ H(G) and S contains a non-zero function then f is a greatest common divisor of S if: (i) f |g for
each g in S and (ii) whenever h|g for each g in S, h| f . In symbols, f = g.c.d.S. Prove that f = g.c.d.S. iff
Z( f ) = Z(S) and show that each non-empty subset of H(G) has a g.c.d.
(c) If A ⊂ G let J(a) = { f ∈ H(G) : Z( f ) ⊃ A}. Show that J(A) is a closed ideal in H(G) and J(A) = (0)

109 110
Solution. We know 7.7 The gamma function
z −z −i(iz) −(−i)(iz)
e −e e −e Exercise 1. Show that 0 < γ < 1. (An approximation to γ is .57722. It is unknown whether γ is rational or
sinh(z) = =
2 2 irrational.)
−ei(iz) + e−i(iz) −ei(iz) + e−i(iz)
= =i Solution. Not available.
2 2i

ei(iz) − e−i(iz) Exercise 2. Show that Γ(z)Γ(1 − z) = π csc πz for z not an integer. Deduce from this that Γ( 12 ) = π.
= −i = −i sin(iz)
2i
∞ ! ∞ ! Solution. Assume z is not an integer, then by Gauss’s Formula p. 178 we get
Y (iz)2 Y z2
= −i(iz) 1− 2 2 =z 1+ 2 2
n=1
nπ n=1
nπ n!nz m!m1−z
Γ(z)Γ(1 − z) = lim lim
z(z + 1) · · · (z + n)
n→∞ m→∞ (1 − z)(2 − z) · · · (m + 1 − z)
since by p. 175 Equation 6.2
n!n!nz+1−z
= lim
∞ ! ∞ ! n→∞ z(1 + z)(1 − z)(2 + z)(2 − z) · · · (n + z)(n − z)(n + 1 − z)
Y z2 Y z2
sin(πz) = πz 1− 2
⇒ sin(z) = z 1− 2 2 . (n!)2 n
n=1
n n=1
nπ = lim
n→∞ z(12 − z2 )(22 − z2 ) · · · (n2 − z2 )(n + 1 − z)
Therefore, (n!)2 n

Y z2
! = lim      
n→∞ z(n!n!) 1 − z2 1 − z2 2 2
1 − 2z 3 · · · 1 − nz 2 (n + 1 − z)
sinh(z) = z 1+ . 12 22
n2 π2
n=1 1
= lim      
We have n→∞z 1− z2
1− z2
1− z2
··· 1 − z2 n+1−z
12 22 32 n2 n
z −z −i(iz) −(−i)(iz) 1
e +e e +e
cosh(z) = = =
2 2 
z2

z2
 2
  
z2 n+1−z
z limn→∞ 1 − 12
1− 22
1 − 3z 2 · · · 1 − lim
n2 n→∞
ei(iz) + e−i(iz) {zn }
= = cos(iz) |
2 =1
∞ ! Y ∞ !
1 π
Y 4(iz)2 4z2 = = Q 
= 1− = 1+ Q  z2 z2

n=1
(2n − 1) 2 π2
n=1
(2n − 1) 2 π2 z ∞ n=1 1 − n2
πz ∞
n=1 1 − n2
π
since by Exercise 1 p. 176 = = π csc(πz)
sin(πz)
∞ ! ∞ !
Y 4z2 Y 4z2 where the step before the last step follows by p. 175 Equation 6.2. So
cos(πz) = 1− ⇒ cos(z) = 1− .
n=1
(2n − 1)2 n=1
(2n − 1) 2 π2
Γ(z)Γ(1 − z) = π csc(πz)
Therefore, 1
∞ ! for z not an integer. Now, let z = 2, then
Y 4z2
cosh(z) = 1+ 2 2
. ! !
n=1
(2n − 1) π 1 1 π
Γ Γ 1− = π csc
    2 2 2
πz πz
Exercise 3. Find a factorization of the function cos 4 − sin 4 .
which implies !
Solution. Not available. 1 √ √
Γ = π· 1 = π.
π Q∞ (2n) 2 2
Exercise 4. Prove Wallis’s formula: 2 = n=1 (2n−1)(2n+1) .

Exercise 3. Show: πΓ(2z) = 22z−1 Γ(z)Γ(z + 21 ). (Hint: Consider the function Γ(z)Γ(z + 21 )Γ(2z)−1 .)
Solution. Not available.
Solution. Following the hint define a function f on its domain G = {z ∈ C : z < (Z − N)} by
!
2z−1  z  z 1
f (z) = √ Γ Γ + .
π 2 2 2
111 112
Let x be a positive real number. It suffices to show that f (x) satisfies the Bohr-Mollerup Theorem. By the Thus,
logarithmic convexity of Γ is follows that ∞ Z ∞
X (−1)n
e−t tz−1 dt
!
√  x x 1 Γ(z) = + (7.12)
log f (x) = (x − 1) log 2 − log π + log Γ + log Γ + n=0
n!(z + n) 1
2 2 2
is a convex function. Also f satisfies the functional equation of the Gamma-function: Claim 1: Γ(z) given by (7.12) is the analytic continuation of (7.11), that is Γ(z) given by (7.12) is defined
! for all z ∈ C − {0, −1, −2, . . .}.
2x x 1 x 
Proof of Claim 1: We know from the book that Ψ(z) is analytic for Re (z) > 0.
f (x + 1) = √ Γ + Γ +1
π 2 2 2 Claim 2: Ψ(z) is analytic for Re (z) ≤ 0. Thus Ψ(z) is analytic on C.
! Proof of Claim 2: Assume Re (z) ≤ 0. Then
2x x 1 x  x
= √ Γ + Γ
π 2 2 2 2
x−1
!   |tz−1 | = tRe(z)−1 ≤ t−1 .
x2 x 1 x t∈[1,∞),Re(z)≤0
= √ Γ + Γ
π 2 2 2
But since e− 2 t tRe (z)−1 → 0 as t → ∞, there exists a constant C > 0 such that tRe(z)−1 ≤ Ce 2 when t ≥ 1.
1 1

= x f (x).
Hence, we have
|e−t tz−1 | ≤ |e−t |· |tz−1 | = e−t tRe(z)−1 ≤ e−t Ce 2 t = Ce− 2 t
1 1
Γ( 1 )
Lastly, also f (1) = √2π = 1; thus by Theorem 7.13, f agrees with Γ on the positive real numbers. With the
1
Identity Theorem then f agrees with Γ on the whole domain G. and therefore Ce− 2 t is integrable on (1, ∞). By Fubini’s Theorem for any {γ} ⊂ G = {z : Re(z) ≤ 0},
Exercise 4. Show that log Γ(z) is defined for z in C − (−∞, 0] and that Z Z ∞ Z ∞Z
e−t tz−1 dt dz = e−t tz−1 dz dt = 0
∞  
X z z γ 1 1 γ
log Γ(z) = − log z − γz − log 1 + − .
n=1
n n which implies Z ∞
Solution. Not available. e−t tz−1 dt ∈ H(G).
1
Exercise 5. Let f be analytic on the right half plane Re z > 0 and satisfy: f (1) = 1, f (z + 1) = z f (z), and In summary,
limn→∞ nf (z+n)
z f (n) = 1 for all z. Show that f = Γ.
Z ∞
Ψ(z) = e−t tz−1 dt ∈ H(C).
Solution. Not available. 1

Exercise 6. Show that Thus, Claim 2 is proved.


∞ Z ∞ It remains to show that
X (−1)n ∞
Γ(z) = + e−t tz−1 dt X (−1)n
n=0
n!(z + n) 1 Φ(z) =
n=0
n!(z + n)
for z , 0, −1, −2, . . . (not for Re z > 0 alone).
is analytic on C − {0, −1, −2, . . .}. Note that Φ(z) is uniformly and absolutely convergent as a series in any
Solution. Write Γ(z) = Φ(z) + Ψ(z) where closed domain which contains none of the points 0, −1, −2, . . . and thus provides the analytic continuation
Z 1 of Φ(z). Since
e−t tz−1 dt ∞ Z ∞
Φ(z) = X (−1)n
0 + e−t tz−1 dt
n=0
n!(z + n) 1
and
Z ∞ is analytic and we know (Theorem 7.15 p. 180)
Ψ(z) = e−t tz−1 dt. (7.11) Z ∞
1
Γ(z) = e−t tz−1 dt
We can write 0
Z 1 Z ∞
1X ∞ Z 1
(−t)n z−1 X (−1)n for Re(z) > 0, we get
Φ(z) = e−t tz−1 dt = t dt = tn tz−1 dt X∞
(−1)n
Z ∞
0 0 n=0
n! n=0
n! 0 Γ(z) = + e−t tz−1 dt
∞ " #1 ∞ n!(z + n) 1
X (−1)n tn+z X (−1)n n=0
= = is the analytic continuation of (7.11) for z ∈ C − {0, −1, −2, . . .}.
n=0
n! n + z 0 n=0
n!(z + n)

113 114
Exercise 7. Show that Z Z r Solution. Let u > 0 and v > 0, then by Theorem 7.15 p. 180 and three changes of variables (indicated by
∞ ∞
2 2 1 1 “CoV”) we obtain
sin(t ) dt = cos(t ) = π.
0 0 2 2 Z ∞ Z ∞
 
1 √ Γ(u)Γ(v) = e−s su−1 ds e−t tv−1 dt
Solution. We have shown that Γ 2 = π (see Exercise 2 p. 185). Thus 0 0
Z ∞ Z ∞
2 2
√ 1
! Z ∞
1
Z ∞
2 2
= 2 e−x x2u−2 x dx· 2 e−y y2v−2 y dy
π=Γ = e−t t− 2 dt = 2a e−a t dt 2
CoV:s=x ,t=y 2
0 0
2 0 0
Z ∞Z ∞
2 2
= 4 e−x −y x2u−1 y2v−1 dx dy
(because of Theorem 7.15 p. 180). The last step involves integrating a complex integral. Hence, we have 0 0
Z π/2 Z ∞
√ 2
e−r r2u−1 (cos θ)2u−1 r2v−1 (sin θ)2v−1 r dr dθ
Z ∞
2 2 π = 4
e−a t dt = . (7.13) CoV:x=r cos(θ),y=r sin θ 0 0
0 2a Z π/2 Z ∞
2
= 4 e−r r2u+2v−1 (cos θ)2u−1 (sin θ)2v−1 r dr dθ
1−i 1−2i+i2
Let a = √ ,
2
then a2 = 2 = −i and therefore 0 0
Z ∞ Z π/2
2
Z ∞ 
1−i
2 Z ∞ Z ∞ = 4 e−r r2u+2v−1 dr (cos θ)2u−1 (sin θ)2v−1 dθ
− √ t2 2 2
e 2
dt = e−(−i)t dt = eit dt 0
Z
0
Z π/2
0 0 0 1 ∞ −t u+v−1
Z ∞ Z ∞ Z ∞ = 4 e t dt (cos θ)2u−1 (sin θ)2v−1 dθ
= cos(t2 ) + i sin(t2 ) dt =
cos(t2 ) dt + i sin(t2 ) dt CoV:t=r 2 2 0 0
Z π/2
0 0 0
√ √ r r r = 2Γ(u + v) (cos θ)2u−1 (sin θ)2v−1 dθ.
π π 1+i 1 1 1 1 1 1 0
= = √ = π(1 + i) = π + i π.
(7.13) 2 1−i
√ 2 (1 − i)(1 + i) 2 2 2 2 2 2
2 This implies
Z π/2
Γ(u)Γ(v)
Thus, r r B(u, v) = =2 (cos θ)2u−1 (sin θ)2v−1 dθ.
Z ∞ Z ∞
1 1 1 1 Γ(u + v) 0
2 2
cos(t ) dt + i sin(t ) dt = π+i π Next, we show
0 0 2 2 2 2 Z 1
which implies r B(u, v) = tu−1 (1 − t)v−1 dt.
Z ∞ 0
2 1 1
cos(t ) dt = π We have
0 2 2
Z π/2
and Γ(u)Γ(v)
Z ∞
1
r
1 B(u, v) = =2 (cos θ)2u−1 (sin θ)2v−1 dθ
sin(t2 ) = π. Γ(u + v) 0
0 2 2 Z 1 
√ 2u−2  √ 2v−2
= 2 t 1−t dt
Exercise 8. Let u > 0 and v > 0 and express Γ(u)Γ(v) as a double integral over the first quadrant of the CoV:t=cos2 θ 0
plane. By changing to polar coordinates show that Z 1
= tu−1 (1 − t)v−1 dt.
Z π 0
2
Γ(u)Γ(v) = 2Γ(u + v) (cos θ)2u−1 (sin θ)2v−1 dθ.
0 Hence, Z 1
The function B(u, v) = tu−1 (1 − t)v−1 dt.
Γ(u)Γ(v) 0
B(u, v) =
Γ(u + v) Finally, we show Z ∞
tu−1
is called the beta function. By changes of variables show that B(u, v) = dt.
0 (1 + t)u+v
Z 1 Z ∞
tu−1
B(u, v) = tu−1 (1 − t)v−1 dt = dt
0 0 (1 + t)u+v
Can this be generalized to the case when u and v are complex numbers with positive real part?

115 116
We have which implies
Z 1
Z 1
tu−1
u−1 v−1
dt ∈ H(G × G).
B(u, v) = t (1 − t) dt 0 (1 + t)u+v
Z0 ∞  II) We have
s u−1  s v−1
= 1− (1 + s)−2 ds Z ∞ Z n
t
CoV:s= 1−t 0 1+s 1+s
Z ∞
|tu−1 (1 + t)−u−v | dt ≤ lim |tu−1 (1 + t)−u−v | dt
1 1 1 n→∞ 1
= su−1 (1 + s)−2 ds Z n
(1 + s)u−1 (1 + s)v−1
0
Z ∞ ≤ lim |tu−1 |· |(1 + t)−u−v | dt
1 n→∞ 1
= su−1 (1 + s)−2 ds Z n
(1 + s)u+v−2
0
Z ∞ = lim tRe(u)−1 (1 + t)−Re(u)−Re(v) dt
su−1 n→∞ 1
= u+v
ds. Z n
1
0 (1 + s) = lim tRe(u)−1  Re(u)+Re(v) dt
n→∞
Hence,
1
tRe(u)+Re(v) 1
t +1
Z ∞
tu−1 Z n
1
B(u, v) =
(1 + t)u+v
dt. = lim t−1−Re(u)  Re(u)+Re(v) dt
0 n→∞ 1 1
t +1
Yes, the beta function can be generalized to the case when u and v are complex numbers with positive real Z n
part. We have seen that 1
≤ lim dt < ∞
Z ∞ Z π/2
n→∞ 1 t1+Re(v)
tu−1 2u−1 2v−1
B(u, v) = dt = 2 (cos θ) (sin θ) dθ where the last step follows since Re(v) > 0 and the previous one since Re(u) + Re(v) > 0, t ≥ 1 and
0 (1 + t)u+v 0
therefore 1
≤ 1. This implies |tu−1 (1 + t)−u−v | is integrable on (1, ∞). By Fubini’s Theorem for
for u > 0 and v > 0. Thus it remains to show that ( 1t +1)Re(u)+Re(v)
any {γ} ⊂ G × G
Z ∞
tu−1 Z Z ∞ Z ∞Z
B(u, v) = u+v
dt ∈ H(G × G) tu−1 tu−1
0 (1 + t) u+v
dt dz = u+v
dz dt = 0
γ 1 (1 + t) 1 γ (1 + t)
where G × G ⊂ C × C with G = {z : Re(z) > 0}.
Let Re(u) > 0 and Re(v) > 0. which implies Z ∞
tu−1
Z ∞ Z 1 Z ∞ dt ∈ H(G × G).
tu−1 tu−1 tu−1 1 (1 + t)u+v
u+v
dt = u+v
dt + u+v
dt .
0 (1 + t) 0 (1 + t) 1 (1 + t) In summary
| {z } | {z } Z ∞
I II tu−1
dt ∈ H(G × G).
0 (1 + t)u+v
I) We have
Z Z Exercise 9. Let αn be the volume of the ball of radius one in Rn (n ≥ 1). Prove by induction and iterated
1 1
integrals that
|tu−1 (1 + t)−u−v | dt ≤ |tu−1 |· |(1 + t)−u−v | dt
0 0 Z 1
Z 1
αn = 2αn−1 (1 − t2 )(n−1)/2 dt
≤ tRe(u)−1 (1 + t)−Re(u)−Re(v) dt 0
0
Z 1 Solution. Define the n-dimensional ball with radius r by
≤ tRe(u)−1 < ∞
0
Bn (r) = {x ∈ Rn : |x| ≤ r}
where the last step follows since Re(u) > 0 and the previous one since Re(u) > 0 and Re(v) > 0 and
4
t ∈ (0, 1). This implies |tu−1 (1 + t)−u−v | is integrable on (0, 1). By Fubini’s Theorem for any {γ} ⊂ G × G and define the volume of Bn (r) by Vn (r). Clearly, V1 (r) = 2r, V2 (r) = πr2 , V3 (r) = 3 πr
3
and therefore
Z Z Z Z V1 (1) = 2, V2 (1) and V3 = 43 π.
1 1
tu−1 tu−1 Claim: For arbitrary n we have
dt dz = dz dt = 0
γ 0 (1 + t)u+v 0 γ (1 + t)u+v Vn (r) = rn Vn (1)

117 118
which implies Exercise 11. The Gaussian psi function is defined by

Vn (r) = rn Vn (1) = rn αn (7.14) Γ0 (z)


(def) Ψ(z) =
Γ(z)
Assume the claim is true, then we write
(a) Show that Ψ is meromorphic in C with simple poles at z = 0, −1, . . . and Res(Ψ; −n) = −1 for n ≥ 0.
Bn (1) = {x ∈ Rn : |x| ≤ 1} = {(x1 , x2 , . . . , xn−1 , t) ∈ Rn : x12 + x22 + . . . + xn−1
2
+ t2 ≤ 1}. (b) Show that Ψ(1) = −γ.
(c) Show that Ψ(z + 1) − Ψ(z) = 1z .
For a fixed t ∈ [−1, 1] we have the balls (d) Show that Ψ(z) − Ψ(1 − z) = −π cot πz.
√  (e) State and prove a characterization of Ψ analogous to the Bohr-Mollerup Theorem.
Bn−1 1 − t2 = {(x1 , x2 , . . . , xn−1 ) ∈ Rn−1 : x12 + x22 + . . . + xn−1
2
≤ 1}
Solution. Not available.
and by (7.14) we obtain
√  √ n−1   n−1 7.8 The Riemann zeta function
Vn−1 1 − t2 = 1 − t2 Vn−1 (1) = 1 − t2 2 αn−1 .
1
√  Exercise 1. Let ξ(z) = z(z − l)π− 2 z ζ(z)Γ( 21 z) and show that ξ is an entire function which satisfies the
Clearly, the ball Bn (1) is the union of all disjoint balls Bn−1 1 − t2 as t varies over [−1, 1]. Hence functional equation ξ(z) = ξ(1 − z).
Z 1   n−1 Z 1   n−1 Solution. The function ξ is analytic on C − ({−2k : k ∈ N0 } ∪ {1}) as a product of analytic functions. The
1 − t2 2 αn−1 dt = 2αn−1 1 − t2 2 dt
1
αn = Vn (1) = factors z, z − 1 and π− 2 z are entire. The zeta-function has a simple pole at z = 1, hence limz→1 (z − 1)ζ(z)
−1 0
exists in C and the function ξ is well-defined at z = 1.  
which proves the formula above. The function Γ 21 z has simple poles at the non-positive even integers. For z = 0, limz→0 zΓ 21 z ∈ C. For
Finally, we prove the claim Vn (r) = rn Vn (1). We have by using a symmetry argument negative even integers z = 2k,  −k ∈ N the poles of Gamma coincide with the simple zeros of the zeta-
function, hence limz→2k ζ(z)Γ 21 z exists in C also. Therefore the function ξ is analytic at each point.
Z Z √2 2Z √2 2 2
r r −x1 Z √ 2 Pn−1 2r −x1 −x2 r − i=1 xi The functional equation of ξ is related to the functional equation of ζ (8.3, p. 192), evaluated at 1 − z, i.e.
Vn (r) = √ 2 2 √ 2 2 2 · · · √ 2 Pn−1 2 1 dxn dxn−1 · · · dx2 dx1 !
−r −
r −x1 − r −x1 −x2 − r − i=1 xi 1
Z r Z √r2 −x2 Z √r2 −x2 −x2 Z √r2 −Pn−1 2
ζ(1 − z) = 2(2π)−z Γ(z)ζ(z) sin π(1 − z) . (7.15)
1 1 2 i=1 xi 2
= 2n ··· 1 dxn dxn−1 · · · dx2 dx1
0 0 0 0 Furthermore the proof uses variants of problems 1, 2 of the homework set 5, in particular
Z Z √ 2Z √ 2 2
r 1−y1 Z √ Pn−1 2
1−y1 −y2 1− i=1 yi
n n ! ! !
= 2 ··· 1· |{z}
r dyn dyn−1 · · · dy2 dy1 1 z 1 z 1
CoV:xi =ryi 0 0 0 0 Γ − Γ + = π csc π(1 − z) (7.16)
Jacobian 2 2 2 2 2
Z 1 Z √1−y2 Z √1−y2 −y2 Z √1−Pn−1
i=1 y
2 √ z 1 z!
= rn
1 1 2
···
i
1 dyn dyn−1 · · · dy2 dy1 πΓ(z) = 2z−1 Γ Γ + . (7.17)
√ √ √ Pn−1 2 2 2
−1 − 1−y21 − 1−y21 −y22 − 1− i=1 y2i

= rn Vn (1).

This implies Vn (r) = rn Vn (1). (Note that this is kind of obvious, since we scale the unit ball in Rn by r in
each dimension to obtain the ball with radius r in Rn ). We could have proved the formula by mathematical
induction. Using spherical coordinates simplifies the computation if preferred.

Exercise 10. Show that


πn/2
αn =
(n/2)Γ(n/2)
where αn is defined in problem 9. Show that if n = 2k, k ≥ 1, then αn = πk /k!

Solution. Not available.

119 120
0
Exercise 7. Prove that ζζ(z)
(z) P∞ Λ(n)
Then the following chain of equalities holds. =− n=1 nz for Re z > 1, where Λ(n) = log p if n = pm for some prime p and
! m ≥ 1; and Λ(n) = 0 otherwise.
1 z 1 z
ξ(1 − z) = (1 − z)(−z)π− 2 + 2 ζ(1 − z)Γ −
2 2 Solution. Not available.
! !
(7.15) 1 z 1 z 1−z −z 1 Exercise 8. (a) Let η(z) = ζ 0 (z)/ζ(z) for Re z > 1 and show that limz→z0 (z − z0 )η(z) is always an integer for
= z(z − 1)π− 2 + 2 Γ − 2 π ζ(z)Γ(z) sin π(1 − z)
2 2 2 Re z0 ≥ 1. Characterize the point z0 (in its relation to ζ) in terms of the sign of this integer.
 
1 Γ(z)Γ 21 − 2z 1
! (b) Show that for  > 0

= ξ(z)π− 2 21−z   sin π(1 − z) X
Γ 2z 2 Re η(1 +  + it) = − Λ(n)n−1(1+) cos(t log n)
  n=1
1 !
(7.16) 1 Γ(z) π csc 2 π(1 − z) 1
= ξ(z)π− 2 21−z     sin π(1 − z) where Λ(n) is defined in Exercise 7.
Γ z
Γ + 1 z 2 (c) Show that for all  > 0,
2 2 2
1
1−z Γ(z)
= ξ(z)π 2 2     3Re η(1 + ) + 4Re η(1 +  + it) + Re η(1 +  + 2it) ≤ 0.
Γ 2z Γ 12 + 2z
(7.17) 1 1 2z−1 1 z
! (d) Show that ζ(z) , 0 if Re z = 1 (or 0).
= ξ(z)π 2 21−z   1 Γ +
1 z
Γ 2 + 2 π2 2 2 Solution. Not available.
= ξ(z).
P
Exercise 2. Use Theorem 8.17 to prove that p−1
n = ∞. Notice that this implies that there are an infinite
number of primes.
Q  1 
Solution. Euler’s Theorem asserts that for real values x > 1, ζ(x) = ∞ n=1 1−p−x
n
, where pn are the prime
numbers. The left hand side is unbounded as x & 1 and so is the right hand side.
P∞ 1
Now the argument goes as follows. Seeking contradiction suppose that n=1 pn is finite. All summands
P  
are positive, therefore the sum converges absolutely. By Proposition 5.4, p.165, also ∞ 1
n=1 log 1 − pn
converges absolutely. But this implies that
∞ ! ∞ !
X 1 Y 1
− log 1 − = log = log ζ(1) < ∞
n=1
pn n=1
1 − pn

which gives the desired contradiction because the log(ζ(z)) has a simple pole at z = 1.
P d(n)
Exercise 3. Prove that ζ 2 (z) = ∞
n=1 nz for Re z > 1, where d(n) is the number of divisors of n.

Solution. Not available.


P∞ σ(n)
Exercise 4. Prove that ζ(z)ζ(z − 1) = n=1 nz , for Re z > 1, where σ(n) is the sum of the divisors of n.
Solution. Not available.
Exercise 5. Prove that ζ(z−1) ϕ(n)
P∞
ζ(z) = n=1 nz for Re z > 1, where ϕ(n) is the number of integers less than n and
which are relatively prime to n.
Solution. Not available.
P µ(n) km
Exercise 6. Prove that ζ(z) 1
= ∞ k1 k2
n=1 nz for Re z > 1, where µ(n) is defined as follows. Let n = p1 p2 · · · pm
be the factorization of n into a product of primes p1 , . . . , pm and suppose that these primes are distinct. Let
µ(1) = 1; if k1 = . . . = km = 1 then let µ(n) = (−1)m ; otherwise let µ(n) = 0.
Solution. Not available.

121 122
a) By the previous exercise, we have seen that u x and uy are harmonic. Therefore, −uy is harmonic, too.
Thus, part a) is proved.
b) We have to show that the Cauchy-Riemann equations are satisfied, that is

(u x ) x = −(uy )y and (u x )y = −(−uy ) x .

Chapter 10 The first is true by (10.2) since


(u x ) x = −(uy )y ⇐⇒ u xx + uyy = 0.
The second is true since u xy = uyx . Therefore

Harmonic Functions (u x )y = −(−uy ) x ⇐⇒ u xy = uyx ⇐⇒ u xy = uyx .

In summary, f = u x − iuy is analytic provided u is harmonic.


P
Exercise 3. Let p(x, y) = nk,l=0 akl xk yl for all x, y in R.
Show that p is harmonic iff:
10.1 Basic properties of harmonic functions (a) k(k − 1)ak,l−2 + l(l − 1)ak−2,l = 0 for 2 ≤ k, l ≤ n;
∂u ∂u (b) an−1,l = an,l = 0 for 2 ≤ l ≤ n;
Exercise 1. Show that if u is harmonic then so are u x = ∂x and uy = ∂y . (c) ak,n−1 = ak,n = 0 for 2 ≤ k ≤ n.
Solution. Let u be harmonic, that is
Solution. Not available.
u xx + uyy = 0. (10.1) Exercise 4. Prove that a harmonic function is an open map. (Hint: Use the fact that the connected subsets
of R are intervals.)
Since u : G → R is harmonic, u is infinitely times differentiable by Proposition 1.3 p. 252. Thus, the third
partial derivatives of u exist and are continuous (therefore u xyy = uyyx and uyxx = u xxy ). Now, we show that Solution. Not available.
u x is harmonic, that is
(u x ) xx + (u x )yy = 0. Exercise 5. If f is analytic on G and f (z) , 0 for any z show that u = log | f | is harmonic on G.

Solution. Not available.


(u x ) xx + (u x )yy = u xxx + u xyy = u xxx + uyyx = (u xx + uyy ) x = 0 x = 0.
(10.1) Exercise 6. Let u be harmonic in G and suppose B̄(a; R) ⊂ G. Show that
Thus, u x is harmonic. Finally, we show that uy is harmonic, that is 1
"
u(a) = u(x, y) dx dy.
(uy ) xx + (uy )yy = 0. πR2 B̄(a;R)

Solution. Let D be a disk such that B̄(a; R) ⊂ D ⊂ G. There exist an f ∈ H(D) such that Re( f ) = u on D.
(uy ) xx + (uy )yy = uyxx + uyyy = u xxy + uyyy = (u xx + uyy )y = 0y = 0. From Cauchy’s Integral Formula, we have
(10.1)
Z Z 2π
1 f (w) 1
Thus, uy is harmonic. f (a) = dw = f (a + reiθ ) dθ
2πi γ w−a 2π 0
Exercise 2. If u is harmonic, show that f = u x − iuy is analytic.
where γ(t) = a + reit , 0 ≤ t ≤ 2π. So
Solution. Since, u is harmonic, we have
Z 2π
1
u xx + uyy = 0. (10.2) f (a) = f (a + reiθ ) dθ.
2π 0

In addition, u is infinitely times differentiable by Proposition 1.3 p. 252. Thus, all the partials are continuous Multiply by r, we get
(therefore u xy = uyx ). 1
Z 2π
To show: f = u x − iuy is analytic which is equivalent to showing f (a)r = f (a + reiθ )r dθ
2π 0
a) Re( f ) = u x and Im( f ) = −uy are harmonic and
b) u x and −uy have to satisfy the Cauchy Riemann equations (by Theorem III 2.29).

137 138
and then integrate with respect to r yields for all θ. We have
 !
Z R Z R
1
Z 2π  1 + reiθ 2 
f (a)r dr = f (a + reiθ )r dθ dr u(reiθ ) = Im  iθ

0 0 2π 0 1 − re 
Z R Z 2π
r2
 ! 
1  1 + reiθ 2 1 − reiθ 2 
!
⇐⇒ f (a) = f (a + reiθ )r dθ dr = Im  
2 2π 0 0 1 − re iθ 1 − re iθ
Z 2π Z R
1  
 (1 + reiθ )(1 − reiθ ) 2 
!
⇐⇒ f (a) = f (a + reiθ )r dr dθ
πR2 0 = Im  
" 0 iθ
(1 − re )(1 − re )  iθ
1
⇐⇒ f (a) = f (x, y) dx dy. "
(1 + reiθ − re−iθ − r2 )2
#
πR2 B̄(a;R) = Im
(1 − reiθ − re−iθ + r2 )2
where the last step follows by changing from polar coordinates to rectangular coordinates. Now " #
(1 + 2ri sin θ − r2 )2
= Im
(1 − 2r cos θ + r2 )2
" "
1 1
u(a) = Re( f (a)) = Re( f (x, y)) dx dy = u(x, y) dx dy. " #
πR2 B̄(a;R) πR2 B̄(a;R) ((1 − r2 ) + 2ri sin θ)2
= Im 2 2
(1 − 2r cos θ + r )
Thus " #
(1 − r2 )2 + 4ri sin θ(1 − r2 ) − 4r2 sin2 θ
"
1
u(a) = u(x, y) dx dy. = Im
πR2 B̄(a;R) (1 − 2r cos θ + r2 )2
Exercise 7. For |z| < 1 let 4r(1 − r2 ) sin θ
 ! = .
 1 + z 2  (1 − 2r cos θ + r2 )2
u(z) = Im   .
1−z  Note that the denominator is zero iff r = 1 and θ = 2πk, k ∈ Z ⇐⇒ z = 1.
If we assume, that r = 1 and θ , 2πk, k ∈ Z, then
Show that u is harmonic and limr→1− u(reiθ ) = 0 for all θ. Does this violate Theorem 1.7? Why?
 2 4· 1· (1 − 12 ) sin θ 0
Solution. Let D = {z ∈ C : |z| < 1}. Let f (z) = 1+z 1−z . Clearly, f ∈ H(D) since z = 1 is not in D. By
lim u(reiθ ) = u(eiθ ) = = = 0.
r→1− (1 − 2· 1· cos θ + 12 )2 22 (1 − cos θ)2
Theorem III 2.29 we obtain that v = Re( f ) and u = Im( f ) are harmonic. Therefore
 !  If θ = 2πk, k ∈ Z, then
 1 + z 2 
u(z) = Im   4r(1 − r2 ) sin(2πk) 0
1−z  lim u(re2πk ) = lim− = lim = 0.
r→1− r→1 (1 − 2r cos(2πk) + r2 )2 r→1− (1 − r)4
is harmonic. Next, we will show
Thus,
lim u(reiθ ) = 0
r→1− lim u(reiθ ) = 0.
r→1−
This does not violate Theorem 1.7. The reason is the following: If we pick u = u and v = 0, then the
condition
lim sup u(z) ≤ 0
z→a

does not hold ∀a ∈ ∂D.


Claim: lim supz→1 u(z) > 0.
Proof of the claim:  !
 1 + x + iy 2  2 2
u(z) = u(x, y) = Im   = 4y(1 − x − y ) .
1 − x − iy  (1 − 2x + x2 + y2 )2
If we fix x = 1, then
4y(−y2 ) −4y3 4
u(1, y) = = 4 =− ∀y , 0.
y4 y y
But − 4y → ∞ as y → 0− .

139 140
Exercise 8. Let u : G → R be a function with continuous second partial derivatives and define U(r, θ) = Solution. Let D = {z : |z| < 1} and suppose f = u + iv : D → C is a continuous function such that u and v
u(r cos θ, r sin θ). are harmonic. Clearly u and v are continuous functions mapping D to R. By Corollary 2.9 we have
(a) Show that " 2 # ! Z π
∂ u ∂2 u 2
∂U ∂2 U ∂2 U 1
2∂ U ∂ ∂U u(reiθ ) = Pr (θ − t)u(eit ) dt for 0 ≤ r < 1 and all θ (10.3)
r2 + = r + r + = r r + 2. 2π −π
∂x2 ∂y2 ∂r2 ∂r ∂θ2 ∂r ∂r ∂θ
So if 0 < G then u is harmonic iff ! and
∂ ∂U ∂2 U Z π
r r + 2 = 0. 1
∂r ∂r ∂θ v(reiθ ) = Pr (θ − t)v(eit ) dt for 0 ≤ r < 1 and all θ. (10.4)
2π −π
(b) Let u have the property that it depends only on |z| and not arg z. That is, u(z) = ϕ(|z|). Show that u is
harmonic iff u(z) = a log |z| + b for some constants a and b. Hence,
Z π Z π
Solution. Not available. 1 1
f (reiθ ) = u(reiθ ) + iv(reiθ ) = Pr (θ − t)u(eit ) dt + i Pr (θ − t)v(eit ) dt
(10.3),(10.4) 2π −π 2π −π
Exercise 9. Let u : G → R be harmonic and let A = {z ∈ G : u x (z) = uy (z) = 0}; that is, A is the set of zeros Z π Z π
1 h i 1
of the gradient of u. Can A have a limit point in G? = Pr (θ − t) u(eit ) + iv(riθ ) dt = Pr (θ − t) f (eit ) dt
2π −π 2π −π
Solution. Let u be harmonic and not constant. By Exercise 1 p. 255, we know that u x and uy are harmonic.
By exercise 2 p. 255, we know that f = u x − iuy is analytic. Define B = {z ∈ G : f (z) = 0} = {z ∈ G : for 0 ≤ r < 1 and all θ. Thus, we have proved
u x (z) − iuy (z) = 0}. Clearly 1
Z π
A = {z ∈ G : u x (z) = uy (z) = 0} = B f (reiθ ) = f (eit )Pr (θ − t) dt
2π −π
since u x (z) − iuy (z) = 0 iff u x (z) = uy (z) = 0 (u x (z) and uy (z) are real). Since f is analytic, the zeros are
isolated and therefore B cannot have a limit point in G (Corollary 3.10 p. 79). This implies that A cannot for all reiθ in D.
have a limit point in G. Next, we will prove Z π
If u is constant, then A = B = G and thus A can have a limit point in G. (In this case f ≡ 0 and by Theorem f is analytic ⇐⇒ f (eiθ )eint dt = 0
3.7 p. 78 A has a limit point in G). −π

Exercise 10. State and prove a Schwarz Reflection Principle for harmonic functions. for all n ≥ 1. ⇒: Let f be analytic on D and assume n ≥ 1 (n integer). Define g(z) = f (z)zn−1 . Clearly g is
analytic on D. Let γ = reit , −π ≤ t ≤ π, 0 < r < 1, then by Cauchy’s Theorem
Solution. Not available. Z
g(z) dz = 0
Exercise 11. Deduce the Maximum Principle for analytic functions from Theorem 1.6. γ
Z
Solution. Not available. ⇐⇒ f (z)zn−1 dz = 0
γ
Z π
10.2 Harmonic functions on a disk ⇐⇒ f (reit )ireit eit(n−1) rn−1 dt = 0
−π
Z π
Exercise 1. Let D = {z : |z| < 1} and suppose that f : D− → C is a continuous function such that both Re f ⇐⇒ i f (reit )eint rn dt = 0
and Im f are harmonic. Show that −π
Z π
1
Z π ⇐⇒ f (reit )eint rn dt = 0
f (reiθ ) = f (eit )Pr (θ − t) dt −π
2π −π
which implies Z Z
π π
for all reiθ in D. Using Definition 2.1 show that f is analytic on D iff lim f (reit )eint rn dt = 0 ⇒ f (eit )eint dt = 0 ∀n ≥ 1.
Z π r→1− −π by Ex 3a p. 262
−π

f (eit )eint dt = 0
−π

for all n ≥ 1.

141 142

⇐: Assume −π
f (reit )eint rn dt = 0 for all n ≥ 1. We have for all z ∈ D Solution. Not available.

1
Z π Exercise 4. Let G be a simply connected region and let Γ be its closure in C∞ ; ∂∞G = Γ − G. Suppose
f (z) = f (reiθ ) = f (eit )Pr (θ − t) dt there is a homeomorphism ϕ of Γ onto D− (D = {z : |z| < 1}) such that ϕ is analytic on G.
Part 1 2π
−π
Z π ∞ (a) Show that ϕ(G) = D and ϕ(∂∞G) = ∂D.
1 X
(b) Show that if f : ∂∞G → R is a continuous function then there is a continuous function u : Γ → R such
= f (eit ) r|n| ein(θ−t) dt
2π −π n=−∞ that u(z) = f (z) for z in ∂∞G and u is harmonic in G.

1 X |n| inθ π
Z (c) Suppose that the function f in part (b) is not assumed to be continuous at ∞. Show that there is a
= r e f (eit )e−int dt continuous function u : G− → R such that u(z) = f (z) for z in ∂G and u is harmonic in G (see Exercise 2).
2π n=−∞ −π
−1 Z ∞ Z Solution. Not available.
1 X |n| inθ π 1 X n inθ π
= r e f (eit )e−int dt + r e f (eit )e−int dt Exercise 5. Let G be an open set, a ∈ G, and G0 = G − {a}. Suppose that u is a harmonic function on G0
2π n=−∞ 2π n=0
|−π {z } −π
such that limz→a u(z) exists and is equal to A. Show that if U : G → R is defined by U(z) = u(z) for z , a
=0 by assumption

∞ Z and U(a) = A then U is harmonic on G.


π
1 X n
= z f (eit )e−int dt. Solution. Not available.
2π n=0 −π
Exercise 6. Let f : {z : Re z = 0} → R be a bounded continuous function and define u : {z : Re z > 0} → R
Now, for any closed rectifiable curve γ in D we get by Z
Z Z ∞ Z ∞ Z Z π 1 ∞ x f (it)
1 X n π 1 X u(x + iy) = dt.
f (z) dz = z f (eit )e−int dt dz = zn dz f (eit )e−int dt π −∞ x2 + (y − t)2
γ γ 2π n=0 −π 2π n=0 γ −π
| {z } Show that u is a bounded harmonic function on the right half plane such that for c in R, f (ic) = limz→ic u(z).
=0
Solution. Not available.
since zn is analytic for n ≥ 0.
Thus f is analytic on D. Exercise 7. Let D = {z : |z| < l} and suppose f : ∂D → R is continuous except for a jump discontinuity at
z = 1. Define u : D → R̄ by (2.5). Show that u is harmonic. Let v be a harmonic conjugate of u. What can
Exercise 2. In the statement of Theorem 2.4 suppose that f is piecewise continuous on ∂D. Is the conclusion you say about the behavior of v(r) as r → 1− ? What about v(reiθ ) as r → 1− and θ → 0?
of the theorem still valid? If not, what parts of the conclusion remain true?
Solution. Not available.
Solution. Not available.
Exercise 3. Let D = {z : |z| < 1}, T = ∂D = {z : |z| = 1} 10.3 Subharmonic and superharmonic functions
(a) Show that if g : D− → C is a continuous function and gr : T → C is defined by gr (z) = g(rz) then
gr (z) → g(z) uniformly for z in T as r → 1− . Exercise 1. Which of the following functions are subharmonic? superharmonic? harmonic? neither
(b) If f : T → C is a continuous function define f˜ : D− → C by f˜(z) = f (z) for z in T and subharmonic nor superharmonic? (a) ϕ(x, y) = x2 + y2 ; (b) ϕ(x, y) = x2 − y2 ; (c) ϕ(x, y) = x2 + y; (d)
Z π ϕ(x, y) = x2 − y; (e) ϕ(x, y) = x + y2 ; (f) ϕ(x, y) = x − y2 .
1
f˜(reiθ ) = f (eit )Pr (θ − t) dt Rπ Rπ Rπ Rπ
2π −π Solution. Note that −π sin θ dθ = 0, −π cos θ dθ = 0, −π sin2 θ dθ = π and −π cos2 θ dθ = π. For all
a = (α, β) ∈ C and any r > 0 we have
(So Re f˜ and Im f˜ are harmonic in D). Define f˜r : T → C by f˜r (z) = f˜(rz). Show that for each r < 1 there a)
is a sequence {pn , (z, z̄)} of polynomials in z and z̄ such that pn (z, z̄) → f˜r (z) uniformly for z in T . (Hint: Use
Definition 2.1.) ϕ(a + reiθ ) = ϕ(α + r cos θ, β + r sin θ) = (α + r cos θ)2 + (β + r sin θ)2
(c) Weierstrass approximation theorem for T. If f : T → C is a continuous function then there is a = α2 + 2αr cos θ + r2 cos2 θ + β2 + 2βr sin θ + r2 sin2 θ
sequence {pn (z, z̄)} of polynomials in z and z̄ such that pn (z, z̄) → f (z) uniformly for z in T .
= α2 + β2 + 2αr cos θ + 2βr sin θ + r2 .
(d) Suppose g : [0, 1] → C is a continuous function such that g(0) = g(1). Use part (c) to show that there is
a sequence {pn } of polynomials such that pn (t) → g(t) uniformly for t in [0, 1]. Thus
(e) Weierstrass approximation theorem for [0,1]. If g : [0, 1] → C is a continuous function then there is a Z π Z π Z
1 αr βr π
sequence {pn } of polynomials such that pn (t) → g(t) uniformly for t in [0, 1]. (Hint: Apply part (d) to the ϕ(a + reiθ ) dθ = α2 + β2 + cos θ dθ + sin θ dθ +r2
2π −π π π
function g(t) + (1 − t)g(1) + tg(0).) |−π {z } |−π {z }
(f) Show that if the function g in part (e) is real valued then the polynomials can be chosen with real =0 =0
coefficients. = α2 + β2 + r2 ≥ α2 + β2 = ϕ(a).

143 144
Hence, ϕ ∈ Subhar(G). Thus
b) Z π Z π Z Z π
1 r βr π r2
ϕ(a + reiθ ) dθ = α + β2 + cos θ dθ + sin θ dθ + sin2 θ dθ
ϕ(a + reiθ ) = ϕ(α + r cos θ, β + r sin θ) = (α + r cos θ)2 − (β + r sin θ)2 2π −π 2π
|−π {z }
π −π
| {z }
2π −π
| {z }
= α2 + 2αr cos θ + r2 cos2 θ − β2 − 2βr sin θ − r2 sin2 θ. =0 =0 =π
r22
Thus = α+β + ≥ α + β2 = ϕ(a).
2
Z π Z π Z π Z
1 αr r2 βr π Hence, ϕ ∈ Subhar(G).
ϕ(a + reiθ ) dθ = α2 − β2 + cos θ dθ + cos2 θ dθ − sin θ dθ
2π −π π 2π π f)
|−π {z } |−π {z } |−π {z }
=0 =π =0
Z π
ϕ(a + reiθ ) = ϕ(α + r cos θ, β + r sin θ) = (α + r cos θ) − (β + r sin θ)2
r2 2 r2 r22 2
− sin θ dθ = α − β + − = α2 − β2 = ϕ(a). = α + r cos θ − β2 − 2βr sin θ − r2 sin2 θ.
2π −π 2 2
| {z }
=π Thus
Z π Z π Z Z π
Hence, ϕ ∈ Har(G), therefore also ϕ ∈ Subhar(G) and ϕ ∈ Superhar(G). 1 r βr π r2
ϕ(a + reiθ ) dθ = α − β2 + cos θ dθ − sin θ dθ − sin2 θ dθ
c) 2π −π 2π π −π 2π −π
|−π {z } | {z } | {z }
iθ 2 =0 =0 =π
ϕ(a + re ) = ϕ(α + r cos θ, β + r sin θ) = (α + r cos θ) + (β + r sin θ)
r2
= α2 + 2αr cos θ + r2 cos2 θ + β + r sin θ. = α − β2 − ≤ α − β2 = ϕ(a).
2
Thus Hence, ϕ ∈ Superhar(G).
Z π Z π Z π Z π
1 iθ 2 αr r2 r Exercise 2. Let Subhar(G) and Superhar(G) denote, respectively, the sets of subharmonic and superhar-
ϕ(a + re ) dθ = α +β+ cos θ dθ + cos2 θ dθ + sin θ dθ
2π −π π 2π 2π monic functions on G.
|−π {z } |−π {z } |−π {z }
=0 =π =0
(a) Show that Subhar(G) and Superhar(G) are closed subsets of C(G; R).
r2 (b) Does a version of Harnack’s Theorem hold for subharmonic and superharmonic functions?
2
= α +β+ ≥ α2 + β = ϕ(a).
2 Solution. Not available.
Hence, ϕ ∈ Subhar(G). Exercise 3. (This exercise is difficult.) If G is a region and if f : ∂∞G → R is a continuous function let
d) u f be the Perron Function associated with f . This defines a map T : (∂∞G; R) → Har(G) by T ( f ) = u f .
Prove:
ϕ(a + reiθ ) = ϕ(α + r cos θ, β + r sin θ) = (α + r cos θ)2 − (β + r sin θ) (a) T is linear (i.e., T (a1 f1 + a2 f2 ) = a1 T ( f1 ) + a2 T ( f2 )).
= α2 + 2αr cos θ + r2 cos2 θ − β − r sin θ. (b) T is positive (i.e., if f (a) ≥ 0 for all a in ∂∞G then T ( f )(z) ≥ 0 for all z in G).
(c) T is continuous. Moreover, if { fn } is a sequence in C(∂∞G; R) such that fn → f uniformly then T ( fn ) →
Thus T ( f ) uniformly on G.
1
Z π
αr
Z π
r2
Z π
r
Z π (d) If the Dirichlet Problem can be solved for G then T is one-one. Is the converse true?
ϕ(a + reiθ ) dθ = α2 − β + cos θ dθ + cos2 θ dθ − sin θ dθ
2π −π π 2π −π 2π −π Solution. Not available.
|−π {z } | {z } | {z }
=0 =π =0
Exercise 4. In the hypothesis of Theorem 3.11, suppose only that f is a bounded function on ∂∞G; prove
r2 that the conclusion remains valid. (This is useful if G is an unbounded region and g is a bounded continuous
= α2 − β + ≥ α2 − β = ϕ(a).
2 function on ∂G. If we define f : ∂∞G → R by f (z) = g(z) for z in ∂G and f (∞) = 0 then the conclusion
Hence, ϕ ∈ Subhar(G). of Theorem 3.11 remains valid. Of course there is no reason to expect that the harmonic function will have
e) predictable behavior near ∞ — we could have assigned any value to f (∞). However, the behavior near
points of ∂G can be studied with hope of success.)
ϕ(a + reiθ ) = ϕ(α + r cos θ, β + r sin θ) = (α + r cos θ) + (β + r sin θ)2
Solution. Not available.
= α + r cos θ + β2 + 2βr sin θ + r2 sin2 θ.
Exercise 5. Show that the requirement that G1 is bounded in Corollary 3.5 is necessary.

145 146
Solution. Not available. 10.4 The Dirichlet Problem
Exercise 6. If f : G → Ω is analytic and ϕ : Ω → R is subharmonic, show that ϕ ◦ f is subharmonic if f Exercise 1. Let G = B(0; 1) and find a barrier for G at each point of the boundary.
is one-one. What happens if f 0 (z) , 0 for all z in G?
Solution. Not available.
Solution. Clearly f is continuous, since f : G → Ω is analytic and ϕ is continuous, since ϕ : Ω → R is
subharmonic. Define u = ϕ ◦ f , then u : G → R is continuous. If we can show that for every bounded Exercise 2. Let G = C − (∞, 0] and construct a barrier for each point of ∂∞G.
region G1 such that Ḡ1 ⊂ G and for every continuous function u1 : Ḡ1 → R that is harmonic in G1 and
satisfies u(z) ≤ u1 (z) for z in ∂G1 , we have Solution. Not available.
u(z) ≤ u1
Exercise 3. Let G be a region and a a point in ∂∞G such that there is a harmonic function u : G → R with
for z in G1 , then u is subharmonic (by Corollary 3.5 p. 265). limz→a u(z) = 0 and lim inf z→w u(z) > 0 for all w in ∂∞G, w , a. Show that there is a barrier for G at a.
Hence, let G1 be a bounded region G1 such that Ḡ1 ⊂ G and let u1 : Ḡ1 → R that is harmonic in G1 and
satisfies u(z) ≤ u1 (z) for z on ∂G1 . We have to show u(z) ≤ u1 (z) ∀z ∈ G1 . Solution. Not available.
Since f is one-one, we know that f −1 exists and is analytic. So define
Exercise 4. This exercise asks for an easier proof of a special case of Theorem 4.9. Let G be a bounded
ϕ1 = u1 ◦ f −1 : Ω̄ → R region and let a ∈ ∂G such that there is a point b with [a, b] ∩ G− = {a}. Show that G has a barrier at a.
(Hint: Consider the transformation (z − a)(z − b)1 .)
with f −1 : Ω̄1 → Ḡ1 . Clearly ϕ1 is harmonic (harmonic composite analytic is harmonic). Since ϕ : Ω → R
is subharmonic, we can use Theorem 3.4 to obtain: For every region Ω1 contained in Ω and every harmonic Solution. Not available.
function ϕ̃ on Ω1 , ϕ−ϕ̃ satisfies the Maximum Principle on Ω1 . In fact ϕ−ϕ1 satisfies the Maximum Principle
on Ω1 . By assumption we have
u(z) ≤ u1 (z) ∀z ∈ G1 10.5 Green’s Function
which is the same as Exercise 1. (a) Let G be a simply connected region, let a ∈ G, and let f : G → D = {z : |z| < 1} be a
u(z) = ϕ( f (z)) ≤ u1 (z) = ϕ1 ( f (z)) ∀z ∈ ∂G1 one-one analytic function such that f (G) = D and f (a) = 0. Show that the Green’s Function on G with
since f maps G1 onto Ω1 , and thus ∂G1 onto ∂Ω1 (we choose this by assumption). Thus, singularity at a is ga (z) = − log | f (z)|.
(b) Find the Green’s Functions for each of the following regions: (i) G = C − (∞, 0]; (ii) G = {z : Re z > 0};
ϕ( f (z)) − ϕ1 ( f (z)) ≤ 0 ∀z ∈ ∂G1 . (iii) G = {z : 0 < Im z < 2π}.
Since ϕ − ϕ1 satisfies the Maximum Principle on Ω1 , this implies Solution. Solution to part a):
ϕ( f (z)) − ϕ1 ( f (z)) ≤ 0 ∀z ∈ G1 , Let G be simply connected, let a ∈ G, and let f : G → D = {z : |z| < 1} be a one-one analytic function such
that f (G) = D and f (a) = 0.
that is To show: ga (z) = − log | f (z)| is a Green’s Function on G with singularity at a. Clearly, ga (z) has a singu-
u(z) − u1 (z) ≤ 0 ∀z ∈ G1 larity if | f (z)| = 0. This happens if f (z) = 0. By assumption f (z) = 0 iff z = a. (Note that there is no other
and we are done. Therefore, u = ϕ ◦ f is subharmonic on G. point z ∈ G such that f (z) = 0 since f is assumed to be one-one). This implies ga (z) has a singularity at a.
It remains to verify a), b) and c) of Definition 5.1.
If f 0 (z) , 0 for all z in G, then the previous result holds locally for some neighborhood in G. ( f is lo- a) ga is harmonic in G − {a}. This follows directly by Exercise 5 p. 255 in G − {a}.
cally one-one and onto and f −1 is analytic and hence u = ϕ ◦ f is locally subharmonic). Here is the claim: b) G(z) = ga (z)+log |z−a| is harmonic in a disk about a. Let the disk be B(a; r). Clearly ga (z) = − log | f (z)| is
Let f : G → C be analytic, let z0 ∈ A, and let f 0 (z0 ) , 0. Then there is a neighborhood U of z0 and a harmonic on B(a; r)−{a}. By Exercise 5 p. 255, we can argue again that log |z−a| is harmonic on B(a; r)−{a}
neighborhood V of w0 = f (z0 ) such that f : U → V is one-one and onto and f −1 : V → U is analytic. (choose f (z) = z − a and f (z) = 0 iff z = a). Hence G(z) = ga (z) + log |z − a| = − log | f (z)| + log |z − a| is
Proof of the claim: f (z) − w0 has a simple zero at z0 since f 0 (z0 ) , 0. We can use Theorem 7.4 p. 98 to harmonic on B(a; r) − {a} as a sum of two harmonic functions. But
find  > 0 and δ > 0 such that each w with |w − w0 | < δ has exactly one pre-image z with |z − z0 | < . Let ! !
| f (z)| |z − a|· | f˜(z)|
V = {w| |w − w0 | < δ} and let U be the inverse image of V under the map f restricted to {z| |z − z0 | < }. G(z) = − log | f (z)| + log |z − a| = − log = − log = − log | f˜(z)|
By Theorem 7.4 p. 98, f maps U one-one onto V. Since f is continuous, U is a neighborhood of z0 . By the |z − a| |z − a|
Open Mapping Theorem (Theorem 7.5 p. 99), f = ( f −1 )−1 is an open map, so f −1 is continuous from V to
where f (z) = (z − a) f˜(z) by assumption and f˜(z) is analytic on G with no zero. So G(z) has a removable
U. To finally show that it is analytic, we can use Proposition 3.7 p. 125. We have
Z singularity at a and therefore G(z) is harmonic on B(a; r).
1 z f 0 (w) c) limz→w ga (z) = 0 for each w in ∂∞G. We have | f (z)| = 1 ∀z ∈ ∂∞G by assumption ( f (∂∞G) = ∂D). Hence
f −1 (w) = dz.
2πi |z−z0 |= f (z) − w
lim ga (z) = lim − log | f (w)| = − log 1 = 0
Note, if we assume that ϕ is twice differentiable, then the result holds globally. z→w w→a

147 148
for each a in ∂∞G. This implies ga (z) = − log | f (z)| is a Green’s Function on G with singularity at a. |w0 | < r, have the same number of solutions in Gr . Prove that f is one-one on Gr . From here conclude that
Solution to part b) ii): f (G) = D = {z : |z| < 1} and f 0 (a) > 0, for a suitable choice of α.
Clearly, G is a simply connected region (not the whole plane). If we can find a map, say h : G → D, then (b) Let G be a simply connected region with G , C, but assume that G is unbounded and 0, ∞ ∈ ∂∞G. Let l
the Green’s Function, say ga , is given by be a branch of log z on G, a ∈ G, and α = l(a). Show that l is one-one on G and l(z) , α + 2πi for any z in
G. Prove that ϕ(z) = [l(z) − α − 2πi]−1 is a conformal map of G onto a bounded simply connected region in
ga (z) = − log |h(z)| the plane. (Show that l omits all values in a neighborhood of α + 2πi.)
(c) Combine parts (a) and (b) to prove the Riemann Mapping Theorem.
by Part 1 a). Note that h has to satisfy the following assumptions to use Part 1 a):
a) h : G → D one-one and analytic Solution. Not available.
b) h(a) = 0
c) h(G) = D. Exercise 4. (a) Let G be a region such that ∂G = γ is a simple continuously differentiable closed curve. If
The existence and uniqueness of this function h with the assumptions a), b) and c) is guaranteed by the f : ∂G → R is continuous and g(z, a) = ga (z) is the Green’s Function on G with singularity at a, show that
Riemann Mapping Theorem p. 160 Theorem 4.2. It remains to find h: Z
∂g
Claim: h : G → D given by h(a) = f (z) (z, a) ds (5.5)
γ ∂n
h(z) = f (g(z))
where is a formula for the solution of the Dirichlet Problem with boundary values f ; where ∂g ∂n is the derivative of
z−1 z − g(a) g in the direction of the outward normal to γ and ds indicates that the integral is with respect to arc length.
g(z) = and f (z) =
z+1 1 − g(a)z (Note: these concepts are not discussed in this book but the formula is sufficiently interesting so as to merit
works. Thus, the Green’s Function for the region G = {z : Re(z) > 0} is given by ga (z) = − log |h(z)| where presentation.) (Hint: Apply Green’s formula
h(z) is given by the Claim. " Z " #
∂v ∂u
Proof of the Claim: [u∆v − v∆u] dx dy = u −v ds
Ω ∂Ω ∂n ∂n
We will invoke the Orientation Principle (p. 53) to find g : G → D where g is one-one and analytic. Then
z−1 with Ω = G − {z : |z − a| ≤ δ}, δ < d(a, {γ}), u = h, v = ga (z) = g(z, a).)
g(z) = (b) Show that if G = {z : |z| < 1} then (5.5) reduces to equation (2.5).
z+1
(see p. 53 in the book for the derivation). Solution. Not available.
From p. 162 we have seen that the Möbius Transformation (which is one-one and analytic)
z − g(a)
f (z) =
1 − g(a)z
maps D onto D such that f (g(a)) = 0. Hence h(z) = f (g(z)) maps G onto D. In addition h is one-one,
analytic and h(a) = 0 and therefore satisfies a), b) and c).
Exercise 2. Let ga be the Green’s Function on a region G with singularity at z = a. Prove that if ψ is a
positive superharmonic function on G − {a} with lim inf z→a [ψ(z) + log |z − a|] > −∞, then ga (z) ≤ ψ(z) for
z , a.
Solution. Not available.
Exercise 3. This exercise gives a proof of the Riemann Mapping Theorem where it is assumed that if G is
a simply connected region, G , C, then: (i) C∞ − G is connected, (ii) every harmonic function on G has a
harmonic conjugate, (iii) if a < G then a branch of log(z − a) can be defined.
(a) Let G be a bounded simply connected region and let a ∈ G; prove that there is a Green’s Function ga on
G with singularity at a. Let u(z) = ga (z) + log |z − a| and let v be the harmonic conjugate of u. If ϕ = u + iv
let f (z) = eiα (z − a)e−ϕ(z) for a real number α. (So f is analytic in G.) Prove that | f (z)| = exp(−ga (z))
and that limz→w | f (z)| = 1 for each w in ∂G (Compare this with Exercise 1). Prove that for 0 < r < 1,
Cr = {z : | f (z)| = r} consists of a finite number of simple closed curves in G (see Exercise VI.1.3). Let Gr
be a component of {z : | f (z)| < r} and apply Rouche’s Theorem to get that f (z) = 0 and f (z) − w0 = 0,

149 150
Therefore
n
Y r
|F(0)| = |g(0)|rm
k=1
|ak |
implies
n n
X r f (m) (0) X r
log |F(0)| = log |g(0)| + m log r + log = log + m log r + log .
Chapter 11 k=1
|ak | m! k=1
|ak|

Hence,
n Z 2π
f (m) (0) X r 1
log |F(0)| = log + m log r + log = log | f (reiθ )| dθ
Entire Functions which is equivalent to
m! k=1
|ak | 2π 0

n Z 2π
f (m) (0) X r 1
log + m log r = − log + log | f (reiθ )| dθ
m! k=1
|ak | 2π 0
11.1 Jensen’s Formula which proves the assertion.

Exercise 1. In the hypothesis of Jensen’s Formula, do not suppose that f (0) , 0. Show that if f has a zero Exercise 2. Let f be an entire function, M(r) = sup{| f (reiθ )| : 0 ≤ θ ≤ 2π}, n(r) = the number of zeros of f
at z = 0 of multiplicity m then in B(0; r) counted according to multiplicity. Suppose that f (0) = 1 and show that n(r) log 2 ≤ log M(2r).
Solution. Since f is an entire function, it is analytic on B̄(0; 2r). Suppose a1 , . . . , an are the zeros of f in
n ! Z 2π
f (m) (0) X r 1 B(0; r) repeated according to multiplicity and b1 , . . . , bm are the zeros of f in B(0; 2r) − B(0; r). That is,
log + m log r = − log + log | f (reiθ )| dθ.
m! k=1
|ak | 2π 0 a1 , . . . , an , b1 , . . . , bm are the zeros of f in B(0; 2r) repeated according to multiplicity. Since f (0) = 1 , 0,
we have by Jensen’s formula
r(z−ak )
Solution. We have r2 −āk z
maps B(0; r) onto itself and takes the boundary to the boundary. Let Xn
2r
! X m
2r
!
1
Z 2π
0 = log 1 = log | f (0)| = − log − log + log | f (2reiθ )| dθ
n |ak | |bk | 2π 0
rm Y r2 − āk z k=1 k=1
F(z) = f (z) m
. which implies
z k=1 r(z − ak )
n ! m ! Z 2π
X 2r X 2r 1
Then F ∈ H(G) and F has no zeros in B(0; r) and |F(z)| = | f (z)| if |z| = r. Thus, by a known result, we have log = − log + log | f (2reiθ )| dθ
k=1
|ak | k=1
|bk | 2π 0
Z 2π Z 2π | {z }
1 1 ≥ 0 since r ≤ |bk | < 2r
log |F(0)| = log |F(reiθ )| dθ = log | f (reiθ )| dθ.
2π 0 2π 0 Z 2π
1
≤ log | f (2reiθ )| dθ
Also 2π 0
Z 2π
P∞ f (i) (0)zi 1
f (z) i=0 i!
≤ log |M(2r)| dθ = log |M(2r)|.
g(z) ≡ = Def of M(r) 2π 0
z m zm
(i)
P∞ f (0)z i Hence,
i=m i! n !
=
X 2r
zm log ≤ log |M(2r)|.
∞ k=1
|ak |
X f (i) (0)zi−m f (m) (0) 0 f (m+1) (0) 1
= = z + z + ... But
i=m
i! m! (m + 1)! n(r) n(r)
n ! ! !
X 2r X 2r Y 2r
(m) log = log = log
( f has a zero at z = 0 of multiplicity m) which implies g(0) = f (0) |ak | |ak | |ak |
m! . Thus, k=1 k=1 k=1
 n(r)  n(r)
n !  Y r    Y 2r
Y r = log 2n  = log 2n(r) + log
F(0) = g(0)rm − . |ak | |ak |
k=1
ak k=1 k=1
≥ n(r) log(2)

151 152
r Qn(r) r f (n) (0)
since 0 < |ak | < r implies |ak | > 1 ∀k which means k=1 |ak | > 1 so the logarithm is greater than zero. Thus, where cn = n! . Hence,

n(r) log(2) ≤ log |M(2r)| M(r) µ+1


|cn | ≤ < eαr . (11.2)
rn (11.1)
which proves the assertion.
1
Exercise 3. In Jensen’s Formula do not suppose that f is analytic in a region containing B̄(0; r) but only Define, r = n µ+1 λ, where λ is a positive constant, and choose λ > e for convenience. Then (11.2) can be
that f is meromorphic with no pole at z = 0. Evaluate written in the form
µ+1 µ+1  αλµ+1 n
eαnλ 1 eαnλ  e 
⇐⇒ (nn ) µ+1 |cn | <
Z 2π
1 |cn | < =   .
log | f (reiθ )| dθ. 1
(nn ) µ+1 λn λ n λ
2π 0
Now, if we choose α such that αλµ+1 = 1, the last inequality becomes
Solution. Not available.
1
 e n
Exercise 4. (a) Using the notation of Exercise 2, prove that (nn ) µ+1 |cn | < <
λ
Z r n !
n(t) X r for any  > 0, provided n is taken large enough, n > N say. Since n! < nn , for n ≥ 2, we have
dt = log
0 t k=1
|ak |
1
(n!) µ+1 |cn | < 
where a1 , . . . , an are the zeros of f in B(0; r).
(b) Let f be meromorphic without a pole at z = 0 and let n(r) be the number of zeros of f in B(0; r) minus for n > N or
the number of poles (each counted according to multiplicity). Evaluate lim cn (n!)1/(µ+1) = 0.
n→∞
Z r
n(t) Exercise 2. Let f1 and f2 be entire functions of finite orders λ1 and λ2 respectively. Show that f = f1 + f2
dt.
0 t has finite order λ and λ ≤ max(λ1 , λ2 ). Show that λ = max(λ1 , λ2 ) if λ1 , λ2 and give an example which
Solution. Not available. shows that λ < max(λ1 , λ2 ) with f , 0.

Exercise 5. Let D = B(0; 1) and suppose that f : D → C is an analytic function which is bounded. Solution. Not available.
P
(a) If {an } are the non-zero zeros of f in D counted according to multiplicity, prove that (1 − |an |) < ∞.
Exercise 3. Suppose f is an entire function and A, B, a are positive constants such that there is a r0 with
(Hint: Use Proposition VII. 5.4).
| f (z)| ≤ exp(A|z|a + B) for |z| > r0 . Show that f is of finite order ≤ a.
(b) If f has a zero at z = 0 of multiplicity m ≥ 0, prove that f (z) = zm B(z) exp(−g(z)) where B is a Blaschke
Product (Exercise VII. 5.4) and g is an analytic function on D with Re g(z) ≥ − log M (M = sup{| f (z)| : Solution. Not available.
|z| < 1}).
Exercise 4. Prove that if f is an entire function of order λ then f 0 also has order λ.
Solution. Not available.
Solution. Let f be an entire function of order λ, then f has power series

11.2 The genus and order of an entire function f (z) =
X
cn zn .
P n n=0
Exercise 1. Let f (z) = cn z be an entire function of finite genus µ; prove that
By Exercise 5 e) p. 286 we have
lim cn (n!)1/(µ+1) = 0. − log |cn | 1
n→∞
α = lim inf = .
n→∞ n log n λ
(Hint: Use Cauchy’s Estimate.)
Differentiating f yields
Solution. Define M(r) = max{ f (reiθ ) : 0 ≤ θ ≤ 2π} = max|z|=r | f (z)|. For sufficiently large r = |z|, we have ∞
X ∞
X
f 0 (z) = ncn zn−1 = (n + 1)cn+1 zn .
µ+1 | {z }
M(r) < eαr (11.1) n=1 n=0
dn

by Theorem 2.6 p. 283. By Cauchy’s estimate p. 73, we have


M(r)
|cn | ≤
rn
153 154
Now, let λ0 be the order of f 0 , then Thus
1 |z| 1
| cos z| ≤ cosh |z| = (e + e−|z| ) ≤ e|z| ≤ e|z| ,
1 − log |dn | − log |(n + 1)cn+1 | 2 2
= lim inf = lim inf
λ0 n→∞ n log n n→∞ n log n and therefore √
− log(n + 1) − log |cn+1 | | cos z| ≤ exp(|z|1/2 ).
= lim inf
n→∞ n log n This implies that the order λ ≤ 21 . Next we want to show that λ = 12 . Seeking contradiction assume that
" #
− log(n + 1) log |cn+1 | (n + 1) log(n + 1) there is  > 0 and r > 0 such that
= lim inf − ·
n→∞ n log n n log n (n + 1) log(n + 1) √ 1



 | cos z| ≤ exp(|z| 2 − ) (11.3)
 
 − log(n + 1) − log |c n+1 | n + 1 log(n + 1)  for all |z| > r.
= lim inf  + · ·  √
n→∞ 
| n log n (n + 1) log(n + 1) |{z} n log(n)  Let z = −n2 for an n ∈ N with n > max{ r, 4, 21/(2) } then in particular (i) |z| > r, (ii) − ln 2 + n > 2n , and
{z } | {z } | {z } 1 1
→0 →λ 1 →1 →1 (iii) 2 > n2 and
1 √ 1 1
= . | f (z)| = | cos
n 1 1
z| = |en + e−n | > en = e− log 2+n > e 2 > exp(n−2 n) = exp(n2( 2 −) ) = exp(|z| 2 − ).
λ 2 2 (ii) (iii)

Hence, λ0 = λ and therefore f 0 also has order λ.


This shows that no  > 0 can be found that satisfies equation (11.3). This implies λ = 12 .
P
Exercise 5. Let f (z) = cn zn be an entire function and define the number α by
Exercise 7. Let f1 and f2 be entire functions of finite order λ1 , λ2 ; show that f = f1 f2 has finite order
− log |cn | λ ≤ max(λ1 , λ2 ).
α = lim inf
n→∞ n log n Solution. Define M(r) = max|z|=r | f (z)|, M1 (r) = max|z|=r | f1 (z)| and M2 (r) = max|z|=r | f2 (z)|. Since f1 and
(a) Show that if f has finite order then α > 0. (Hint: If the order of f is λ and β > λ show that |cn | ≤ f2 are entire functions of finite order λ1 and λ2 , respectively, we have for  > 0,
r−n exp(rβ ) for sufficiently large r, and find the maximum value of this expression.) λ1 +/2

(b) Suppose that 0 < α < ∞ and show that for any  > 0,  < α, there is an integer p such that |cn |1/n < M1 (r) < er (11.4)
n−(α−) for n > p. Conclude that for |z| = r > 1 there is a constant A such that and
∞ 
X r n M2 (r) < er
λ2 +/2
(11.5)
| f (z)| < Ar p + α−
n=1
n
for sufficiently large r = |z| by Proposition 2.14. Hence
(c) Let p be as in part (b) and let N be the largest integer ≤ (2r)1/(α−) . Take r sufficiently large so that
N > p and show that M(r) = max | f (z)| = max | f1 (z)· f2 (z)|
|z|=r |z|=r

∞  N  ≤ max | f1 (z)|· max | f2 (z)|


X r n X r n   |z|=r |z|=r
< 1 and < B exp (2r)1/(α−) λ1 +/2 λ2 +/2

n=N+1
nα−
n=p+1
nα−
= M1 (r)M2 (r) < er er
by (11.4),(11.5)
λ1 +/2
+rλ2 +/2 λ+/2 λ+
where B is a constant which does not depend on r. = er ≤ e2r < er
λ≤max(λ1 ,λ2 )
(d) Use parts (b) and (c) to show that if 0 < α < ∞ then f has finite order λ and λ ≤ α−1 .
(e) Prove that f is of finite order iff α > 0, and if f has order λ then λ = α−1 . provided r is sufficiently large. Hence, the order of f (z) = f1 (z)· f2 (z) has finite order λ ≤ max(λ1 , λ2 ).
P
Solution. Not available. Exercise 8. Let {an } be a sequence of non-zero complex numbers. Let ρ = inf{a : |an |−a < ∞}; the
√ number called the exponent of convergence of {an }.
Exercise 6. Find the order of each of the following functions: (a) sin z; (b) cos z; (c) cosh z; (d) (a) If f is an entire function of rank p then the exponent of convergence ρ of the non-zero zeros of f satisfies:
P∞ −an n
n=1 n z where a > 0. (Hint: For part (d) use Exercise 5.) p ≤ ρ ≤ p + 1.
P P
Solution. c) The order is λ = 21 . Note that sinh2 z = cosh2 z − 1 and that cos x is bounded for real values x, (b) If ρ0 = the exponent of convergence of {an } then for every  > 0, |an |−(ρ+) < ∞ and |an |−(ρ−) = ∞.
so (c) Let f be an entire function of order λ and let {a1 , a2 , . . .} be the non-zero zeros of f counted according
| cos z|2 = cos2 x + sinh2 y ≤ cosh2 y ≤ cosh2 |z|. to multiplicity. If ρ is the exponent of convergence of {an } prove that ρ ≤ λ. (Hint: See the proof of (3.5) in
the next section.)

155 156
(d) Let P(z) = ∞
Q
n=1 E p (z/an ) be a canonical product of rank p, and let ρ be the exponent of convergence of 11.3 Hadamard Factorization Theorem
{an }. Prove that the order of P is ρ. (Hint: If λ is the order of P, ρ ≤ λ; assume that |al | ≤ |a2 | ≤ . . . and fix
z, |z| > 1. Choose N such that |an | ≤ 2|z| if n ≤ N and |an | > 2|z| if n ≤ N + 1. Treating the cases ρ < p + 1 Exercise 1. Let f be analytic in a region G and suppose that f is not identically zero. Let G0 = G − {z :
f0
and ρ = p + 1 separately, use (2.7) to show that for some  > 0 f (z) = 0} and define h : G0 → R by h(z) = log | f (z)|. Show that ∂h ∂h
∂x − i ∂y = f on G 0 .


X z
! Solution. Let f be analytic in a region G and suppose that f is not identically 0
zero. Let G0 = G − {z :
log E p < A|z|ρ+ . f (z) = 0}, then h : G0 → R given by h(z) = log | f (z)| is well defined as well as ff is well defined on G0 .
an
n=N+1 Let f = u(x, y) + iv(x, y) = u + iv. Since f is analytic, the Cauchy-Riemann (C-R) equations u x = vy and
1 uy = −v x are satisfied. We have by p. 41 Equation 2.22 and 2.23
Prove that for |z| ≥ 2, log |E p (z)| < B|z| p where B is a constant independent of z. Use this to prove that
f 0 = u x + iv x and f 0 = −iuy + vy
N !
X z ρ+
log E p < C|z| and thus
n=1
an

for some constant C independent of z.) 2 f 0 = u x + iv x − iuy + vy

Solution. Not available. implies

Exercise 9. Find the order of the following entire functions: 1  1  1 


f0 = u x + iv x − iuy + vy = u x − iuy − iuy + u x = 2u x − 2iuy = u x − iuy .
(a) 2 (C-R) 2 2

Y
f (z) = (1 − an z), 0 < |a| < 1 Therefore,
n=1
f 0 u x − iuy
b) = . (11.6)
∞  f u + iv
Y z
f (z) = 1−
n=1
n! Next, we calculate ∂h
= h x and ∂h
= hy where h(z) = log | f (z)| = 1
log(u2 + v2 ). Using the chain rule, we
∂x ∂y 2
c) get
u v
f (z) = [Γ(z)]−1 . h x = hu u x + hv v x =
u2 + v2
ux + 2
u + v2
vx
Q∞  
Solution. b) Considering g(z) := m=1 1 − n!z one notes that g(z) is already in form of the canonical and
u v
product with the entire function in the exponent being constantly zero and p = 0. Hence also the genus hy = hu uy + hv vy = uy + 2 vy
u2 + v2 u + v2
µ = 0. The simple zeros of g(z) are at z = n! and from Exercise 8a) it follows that λ ≤ 1. In fact we will
show that λ = 0. It suffices to show that the exponent of convergence ρ = 0 and to employ Exercise 8d). and hence
Let m ≤ N, let n > 4m then ∂h ∂h
−i = h x − ihy
∂x ∂y
n! ≥ n· (n − 1)· 2· (n − 2)· 3 . . . (n − (2k − 1))· 2m . u v u v
| {z } | {z } |
≥n ≥n
{z
≥n
} = ux + 2 vx − i 2 uy − i 2 vy
u2 + v2 u + v2 u + v2 u + v2
u v
Each product of consecutive numbers is of the form (n − (k − 1))k, for 1 ≤ k ≤ 2m. By choice of n, nk ≤ 12 = (u x − iuy ) + 2 (v x − ivy )
u2 + v2 u + v2
and n − k + 1 > 2m which justifies the individual estimates by n. In total this yields n! > n2m for n large, or u v
equivalently = (u x − iuy ) + 2 (−uy − iu x )
(C-R) u2 + v2 u + v2
X∞ X∞
1 u iv
(n!)− m ≤ n−2 = (u x − iuy ) − 2 (u x − iuy )
n=4m n=4m
u2 + v2 u + v2
u − iv
and the sum on the left converges for every positive integer m. We conclude that ρ = inf{m : the left sum converges} = = (u x − iuy )
u2 + v2
0. By Part d) of Exercise 8 we conclude that the order of g(z) is zero. u − iv
= (u x − iuy )
(u − iv)(u + iv)
u x − iuy
= .
u + iv

157 158
Compare this with (11.6) and we obtain the assertion Therefore by the comparison test, we also have
0
∂h ∂h f ∞
X ∞
X
−i = . |an |−(λ+) ≤ |an |−(ρ+) < ∞.
∂x ∂y f ρ≤λ
n=1 n=1
Exercise 2. Refer to Exercise 2.7 and show that if λ1 , λ2 then λ = max(λ1 ; λ2 ). P P
Hence ∞ n=1 |an |
−(λ+)
< ∞ contradicting the fact ∞n=1 |an |
−(λ+)
= ∞. Thus, F ≡ 0 and therefore we get the
Solution. Not available. assertion f = g.
Exercise 3. (a) Let f and g be entire functions of finite order λ and suppose that f (an ) = g(an ) for a
P
sequence {an } such that |an |−(λ+1) = ∞. Show that f = g.
P
(b) Use Exercise 2.8 to show that if f , g and {an } are as in part (a) with |an |−(λ+) = ∞ for some  > 0
then f = g.
(c) Find all entire functions f of finite order such that f (log n) = n.
(d) Give an example of an entire function with zeros {log 2, log 3, . . .} and no other zeros.
Solution. a) Since f and g are entire functions of finite order λ, also F = f − g is an entire function of finite
P
order λ (see Exercise 2 p. 286). Suppose that f (an ) = g(an ) for a sequence {an } such that |an |−(λ+1) = ∞.
Assume F . 0 ( f . g), we will derive a contradiction. Since F . 0, the sequence {an } are the zeros of F,
because
F(an ) = f (an ) − g(an ) = 0
since f (an ) = g(an ) by assumption. By Hadamard’s Factorization Theorem (p. 289), F has finite genus
µ ≤ λ since F ∈ H(C) with finite order λ. Since by definition µ = max(p, q) where p is the rank of F, we
certainly have
p ≤ λ.
By the definition of the rank (p. 281 Definition 2.1), we have

X
|an |−(p+1) < ∞,
n=1

where an ’s are the zeros of F. Therefore by the comparison test, we also have

X ∞
X
|an |−(λ+1) ≤ |an |−(p+1) < ∞
p≤λ
n=1 n=1
P P
so ∞ n=1 |an |
−(λ+1)
< ∞ contradicting the assumption ∞ n=1 |an |
−(λ+1)
= ∞. Hence F ≡ 0, and therefore we
obtain f = g.
b) Since f and g are entire functions of finite order λ, we also have F = f − g is an entire function of finite
P
order λ (see Exercise 2 p. 286). Suppose that f (an ) = g(an ) for a sequence {an } such that |an |−(λ+) = ∞.
Assume F . 0 ( f . g), we will derive a contradiction. Since F . 0, the sequence {an } are the zeros of F,
because
F(an ) = f (an ) − g(an ) = 0
since f (an ) = g(an ) by assumption. Let ρ be the exponent of convergence of {an }, then

ρ≤λ

by Exercise 8 c) p. 286-287. Hence, according to Exercise 8 b) p. 286-287, we have



X
|an |−(ρ+) < ∞.
n=1

159 160
12.2 The Little Picard Theorem
Exercise 1. Show that if f is a meromorphic function on C such that C∞ − f (C) has at least three points
then f is a constant. (Hint: What if ∞ , f (C)?)

Solution. First consider an entire function f that misses three points in C∞ , one on which is ∞. Then there
are two distinct numbers a, b ∈ C that are not in the image of f . By Little Picard’s Theorem the function f
Chapter 12 must be a constant.
Next consider a meromorphic function on C that is not entire. Hence ∞ ∈ f (C) and by assumption there
are distinct a, b, c ∈ C − f (C). Define a function

The Range of an Analytic Function g(z) :=


1
f (z) − a
, z∈C

1 1
then g is entire because f (z) − a , 0 for all z ∈ C and b−a , c−a ∈ C − g(C). Again using Little Picard’s
Theorem conclude that g is a constant function. It cannot be the zero-function, because f was assumed
12.1 Bloch’s Theorem to be meromorphic, hence f (z) , ∞ for at least some z ∈ C. Thus g is a nonzero constant function and
f (z) = g(z)−1 + a is also a constant function and the result is established.
Exercise 1. Examine the proof of Bloch’s Theorem to prove that L ≥ 1/24.
Exercise 2. For each integer n ≥ 1 determine all meromorphic functions f and g on C with a pole at ∞
Solution. Mimic the proof of Block’s Theorem up to the application of Schwarz’s Lemma, i.e. set K(r) = such that f n + gn = 1.
max{| f 0 (z)|||z| = r}, h(r) = (1 − r)K(r), r0 = sup{r : h(r) = 1}. Choose a with |a| = r0 , | f 0 (a)| = K(r0 ) = 1−r
1
0
.
Solution. Not available.
Also choose ρ0 = 12 (1 − r0 ) and infer that for all z ∈ B(a, ρ)

1 3|z − a|
| f 0 (z)| ≤ , and | f 0 (z) − f 0 (a)| < . 12.3 Schottky’s Theorem
ρ0 2ρ20
No exercises are assigned in this section.
For z ∈ B(0, ρ0 ) define g(z) = f (z + a) − f (a). By convexity of B(a, ρ0 ) the line segment γ = [a, a + z] is
completely contained in B(a, ρ0 ). It follows that
Z
1
12.4 The Great Picard Theorem
|g(z)| = | γ f 0 (w) dw| ≤ |z| ≤ 1 =: M.
ρ0 Exercise 1. Let f be analytic in G = B(0; R) − {0} and discuss all possible values of the integral
The function g is not necessarily injective on B(0, ρ0 ) but the problem only asks for an estimate on the 1
Z
f 0 (z)
Landau constant for which injectivity is not required. In this setting Lemma 1.2 gives dz
2πi γ f (z) − a
! ! !
ρ2 |g0 (0)|2 1 1
g(B(0, ρ0 )) ⊃ B 0, = B 0, = B 0, . where γ is the circle |z| = r < R and a is any complex number. If it is assumed that this integral takes on
6M 4·6 24 certain values for certain numbers a, does this imply anything about the nature of the singularity at z = 0?
This statement rewritten for f yields Solution. Not available.
!
1 Exercise 2. Show that if f is a one-one entire function then f (z) = az + b for some constants a and b, a , 0.
f (B(a, ρ0 )) ⊃ B f (a), .
24
Solution. Not available.
Exercise 2. Suppose that in the statement of Bloch’s Theorem it is only assumed that f is analytic on D.
What conclusion can be drawn? (Hint: Consider the functions f s (z) = s−1 f (sz), 0 < s < 1.) Do the same Exercise 3. Prove that the closure of the set F in Theorem 4.1 equals F together with the constant functions
for Proposition 1.10. ∞, 0, and 1.

Solution. Not available. Solution. Not available.

161 162

You might also like